You are on page 1of 45

www.civilstaphimachal.

com Contact: 7814622609

/.4Subject Number of questions


Current Affairs 20
History 13
Geography 9
Economy 10
Polity 12
Science 8
Environment 8
HPGK 20

Type/Level of questions Number of questions


Easy 21
Medium 41
Tough 38
www.civilstaphimachal.com Contact: 7814622609

Current Affairs- Difficult


1. Consider the following statements regarding National Exit Test (NExT):
1. The National Exit Test (NExT) will be held in 2025.
2. The National Exit Test will replace the final-year MBBS examination and the National Eligibility-cum-Entrance
Test-Postgraduate (NEET-PG), acting as the test for registration for doctors and admission to medical
postgraduate courses.
Which of the following statements is/ are correct?
(a) 1 only
(b) 2 only
(c) Both 1 and 2
(d) Neither 1 nor 2

ANSWER: B
• Union Health Ministry announced that the National Exit Test (NExT) will be held in 2024.
About National Exit Test (NExT)
• It will replace the final-year MBBS examination and the National Eligibility-cum-Entrance Test-Postgraduate
(NEET-PG), acting as the test for registration for doctors and admission to medical postgraduate courses.
• The combined NExT exam will also serve as the mandatory licentiate examination for undergraduate medical
students to practice modern medicine in India.
Objectives
• The importance of the NExt lies in the fact that it will be the same for everyone whether trained in India or
any part of the world and hence it will solve the problem of foreign medical graduates (FMGs) and mutual
recognition.

Current Affairs- Medium


2. Consider the following statements:
1. Environmental Information Awareness Capacity Building and Livelihood Programme is under the Ministry of
Environment, Forests and Climate Change.
2. Mission LiFE (Lifestyle for Environment) was first proposed by the Indian PM at COP 26 (Glasgow).
Which of the following statements is/ are correct?
(a) 1 only
(b) 2 only
(c) Both 1 and 2
(d) Neither 1 nor 2

ANSWER: C
• EIACP Programme Centre at Raipur has created a world record by taking a LiFE (Lifestyle for Environment)
pledge of over 12.38 lakh by people to conserve environment.
About EIACP
• It is a project funded by Ministry of Environment, Forests and Climate Change (MoEF&CC).
• It is one of the Central Sector sub-scheme being implemented in alignment with Mission LiFE.
• The Environmental Information System (ENVIS) is renamed EIACP (Environmental Information, Awareness,
Capacity Building and Livelihood Programme).
o ENVIS came into existence as a planned programme in 1983.
• ENVIS EIACP serves as a one-stop platform for the dissemination of environmental information, informed
policy formulation on various facets of the environment and facilitation of alternate livelihoods through
green skilling.
• It is dedicated to raising awareness, building capacity, and promoting sustainable actions among individuals
and communities across India.
About Mission LiFE (Lifestyle for Environment)
• It was first proposed by the Indian PM at COP 26 (Glasgow).
www.civilstaphimachal.com Contact: 7814622609

• It is envisioned as an India-led global mass movement that will nudge individual and collective action to
protect and preserve the environment.
• It emboldens the spirit of the P3 model, i.e. Pro Planet People.
• It functions on the basic principles of ‘Lifestyle of the planet, for the planet and by the planet’.

Current Affairs- Easy


3. Who has been conferred with the prestigious Padma Shri award for his distinguished service in the field
of agriculture from Himachal Pradesh?
(a) Nek Ram Sharma
(b) Dayal Singh
(c) Anurag Dhumal
(d) None of the above

ANSWER: A
• Nek Ram Sharma, a farmer of Himachal Pradesh's Mandi district, has been conferred with the prestigious
Padma Shri award for his distinguished service in the field of agriculture, especially organic farming.
• Sharma, the only Padma awardee from Himachal Pradesh, has expressed his gratitude to the government
for this achievement.
• He has been growing nine different grains through organic farming.
• Nek Ram said there is a high need to shun the use of fertilizers in farming.
• Praising the 'International Year of Millets 2023', Sharma said, "People have become aware after the
announcement of the International Year of Millets. Earlier there were fewer diseases when people used to
get food products directly from the jungles. Now, in order to earn money, people are involved in unethical
practices which are causing a lot of diseases in the farm produce." The Government of India spearheaded
the United Nations General Assembly (UNGA) resolution for declaring the year 2023 as the 'International
Year of Millets' and the proposal of India was supported by 72 countries.
• UNGA declared the year 2023 as the International Year of Millets in March 2021. January 2023 is the
focused-month for the Ministry of Sports and Youth Affairs, and the states of Chhattisgarh, Mizoram, and
Rajasthan for conducting events and activities related to the millets year.

Current Affairs- Difficult


4. Consider the following statements:
1. Indira Gandhi National Centre for the Arts (IGNCA) is an autonomous institution under the Union Ministry
of Home affairs.
2. Global Biodiversity Framework is a new framework that will guide actions worldwide through 2030, to
preserve and protect nature and its essential services to people.
3. The Geographical Indications of Goods (Registration & Protection) Act, 1999 has come into force in 2005.
Which of the following statements is/are incorrect?
(a) 1 & 2 only
(b) 1 & 3 only
(c) 3 only
(d) 1, 2 and 3

ANSWER: B
• Indira Gandhi National Centre for the Arts (IGNCA) is an autonomous institution under the Union Ministry
of Culture, Government of India.
• The organization serves as a major resource centre for the arts, especially written, oral and visual source
materials.
• It also undertakes research and publication programmes of reference works, glossaries, dictionaries and
encyclopaedia concerning the arts and the humanities.
• The IGNCA initiative on “mera gaon meri dharohar” is related to mapping of information at village levels
related to modern and traditional practices.
www.civilstaphimachal.com Contact: 7814622609

Global Biodiversity Framework (GBF).


• It is a new framework that will guide actions worldwide through 2030, to preserve and protect nature and
its essential services to people
• A geographical indication (GI) is a sign used on products that have a specific geographical origin and possess
qualities or a reputation that are due to that origin.
• India, as a member of the World Trade Organization (WTO), enacted the Geographical Indications of Goods
(Registration & Protection)Act, 1999 has come into force with effect from 15th September 2003.
• GI Tag is basically given to the products related to agriculture, handicrafts, foodstuffs, spirit drinks and
Industrial Products.

Current Affairs- Medium


5. Consider the following statements:
1. Sir David Alan Chipperfield has been selected as the 2023 Laureate of The Pritzker Prize.
2. It is regarded internationally as literature’s highest honor.
Which of the following statements is/ are correct?
(a) 1 only
(b) 2 only
(c) Both 1 and 2
(d) Neither 1 nor 2

ANSWER: A
• Civic architect, urban planner and activist, Sir David Alan Chipperfield has been selected as the 2023
Laureate of The Pritzker Architecture Prize, the award that is regarded internationally as architecture’s
highest honor.
• Chipperfield’s career spans more than 40 years and includes 100-plus projects, ranging from civic, cultural,
and academic buildings to residences and urban master planning throughout Asia, Europe, and North
America.

Current Affairs- Medium


6. Consider the following statements:
1. Sansad Ratna Awards are given by Ministry of Home Affairs.
2. T K Rangarajan will be awarded with Dr APJ Abdul Kalam Lifetime Achievement Award under Sansad Ratna
Awards.
Which of the following statements is/ are correct?
(a) 1 only
(b) 2 only
(c) Both 1 and 2
(d) Neither 1 nor 2

ANSWER: B
• Prime Minister recently congratulated Members of Parliament who will be conferred the Sansad Ratna
Awards 2023.
• The Jury Committee has chosen a total of 13 MPs and two parliamentary committees for the award, with a
lifetime award being presented for the first time this year.
• The Sansad Ratna Awards were instituted in 2010, inspired by the teachings of former President APJ Abdul
Kalam, who launched the first edition of the Award function in Chennai. They seek to recognise and felicitate
the top-performing MPs on the basis of their work in the apex legislative body.
o So far, 90 Parliamentarians have been given this award.
• The Sansad Ratna Awards are not given by the Government of India but by The Prime Point Foundation.
• The jury committee is composed of eminent Parliamentarians and members of civil society.
www.civilstaphimachal.com Contact: 7814622609

• The nominations were based on an MP’s cumulative performance in Parliament, from the beginning of the
17th Lok Sabha until the end of Winter Session 2022.
• Factors that the decision is based on include- questions asked, private members’ Bills introduced, debates
initiated, attendance, funds utilised, etc.
List of Sansad Ratna Awards 2023
• Sansad Ratna Awards 2023 nominated from Lok Sabha
• Adhir Ranjan Chowdhury (INC, West Bengal),
• Gopal Chinayya Shetty (BJP, Maharashtra),
• Sudhir Gupta (BJP, Madhya Pradesh) and
• Dr. Amol Ramsing Kolhe (NCP, Maharashtra)
• Bidyut Baran Mahato (BJP, Jharkhand),
• Dr. Sukanta Majumdar (BJP, West Bengal),
• Kuldeep Rai Sharma (INC, Andaman Nicobar Islands),
• Dr Heena Vijayakumar Gavit (BJP, Maharashtra),

Sansad Ratna Awards 2023 from Rajya Sabha


• Smt Fauzia Tahseen Ahmed Khan (NCP, Maharashtra)
• Dr. John Brittas (CPI-M, Kerala),
• Dr. Manoj Kumar Jha (RJD, Bihar),
• Vishambhar Prasad Nishad (Samajwadi Party, UP) and
• Smt Chhaya Verma (INC, Chhattisgarh)
• Two Parliamentary Committees of Lok Sabha nominated for Sansad Ratna Award 2023
• Rajya Sabha’s Standing Committee on Tourism, Transport, and Culture, chaired by Vijay Sai Reddy
• Lok Sabha’s Parliamentary Committee on Finance, chaired by Jayant Sinha
• Lifetime Achievement Award
• Dr APJ Abdul Kalam Lifetime Achievement Award: T K Rangarajan (Former Rajya Sabha MP for two terms
and a Senior CPIM Leader)

Current Affairs- Medium


7. Consider the following statements:
1. UN Environment Programme (UNEP) and S&P Global have launched Nature Risk Profile.
2. Its methodology is based on Kunming-Montreal Global Biodiversity Framework.
Which of the following statements is/ are correct?
(a) 1 only
(b) 2 only
(c) Both 1 and 2
(d) Neither 1 nor 2

ANSWER: C
• UN Environment Programme (UNEP) and S&P Global have launched Nature Risk Profile.
• NRP is aimed at enabling financial sector to measure and address nature-related risk by providing
scientifically robust and actionable analytics on nature impacts and dependencies.
• Its methodology is based on Kunming-Montreal Global Biodiversity Framework (GBF), adopted in December
2022.
• GBF includes a target for governments to take legal, administrative or policy measures to encourage and
enable businesses to regularly monitor, assess, and transparently disclose their risks, dependencies and
impacts on biodiversity.

Current Affairs- Easy


8. FITUR is one of the world’s largest international travel exhibitions being held at
(a) Madrid
(b) Istanbul
www.civilstaphimachal.com Contact: 7814622609

(c) Los Angeles


(d) Moscow

ANSWER: A
• FITUR is one of the world’s largest international travel exhibitions at Madrid
• Ministry of Tourism, Government of India is participating in FITUR from 18th to 22nd January in Madrid
which is one of the world’s largest international travel exhibitions.
• India’s participation at FITUR is significant to step up the recovery of inbound tourism to pre-pandemic
levels.
• Ministry of Tourism has taken a space of 253 sq. mtr., with over 30 participants co-exhibiting with the India
Pavilion to participate in the Exhibition.
• Incredible India organized various Culture Performances, Yoga Sessions, Mehndi and Bollywood Dance
Performances.

Current Affairs- Medium


9. Consider the following statements about FIFA World Cup:
1. Argentina has won their 3rd FIFA World Cup in 2022.
2. Golden Ball Award is won by Lionel Messi.
Which of the following statements is/are correct?
(a) 1 only
(b) 2 only
(c) Both 1 and 2
(d) Neither 1 nor 2

ANSWER: C
• Argentina clinched their 3rd World Cup trophy from six final appearances in history, seeing off France 4–2
on penalties to win the biggest prize in men’s football at the at Lusail Stadium in Lusail, Qatar.
• It had won two in 1978 and 1986 under Diego Maradona.
• The 2026 World Cup will be held across three different countries, the United States, Canada and Mexico.
2022 FIFA World Cup Awards:
• Top 4 team Rankings: Argentina (Winner), France (2nd), Croatia (3rd) and Morocco (4th)
• Golden Boot Award: Kylian Mbappe (France); (8 Goals)
• Golden Ball Award: Lionel Messi (Argentina); (7 goals and registering 3 assists)
• Golden Glove Award: Emiliano Martinez (Argentina)
• FIFA Young Player Award: Enzo Fernandez Enzo Fernandez
• FIFA Fair Play Award: England
• Silver Boot Award: Lionel Messi
• Bronze Boot Award: Olivier Giroud (France)
• Silver Ball Award: Kylian Mbappe

Current Affairs- Medium


10. Consider the following statements regarding report on First Census of water bodies:
1. 97.1% of total water bodies enumerated, are in rural areas.
2. West Bengal’s South 24 Parganas district has highest number of waterbodies.
Which of the following statements is/ are correct?
(a) 1 only
(b) 2 only
(c) Both 1 and 2
(d) Neither 1 nor 2

ANSWER: C
www.civilstaphimachal.com Contact: 7814622609

• The Ministry of Jal Shakti recently released the report of its first-ever census of water bodies, revealing
crucial insights into the country's water resources.
• Objective- to develop a national database for all water bodies by collecting information on subject including
their size, condition, status of encroachments, use, storage capacity, status of filling up of storage etc.
• Census was launched in convergence with 6th Minor Irrigation Census under Centrally Sponsored Scheme
“Irrigation Census”.
• Water bodies include all natural or man-made units bounded on all sides with some or no masonry work
used for storing water for irrigation or other purposes.
• Findings
o 24.24 lakh water bodies have been enumerated, out of which 97.1% are in rural areas and only 2.9% are
in urban areas.
o West Bengal, Uttar Pradesh, Andhra Pradesh are top 3 states with most water bodies.
o West Bengal’s South 24 Parganas district has highest number of waterbodies.
o States/UTs with least water bodies: Sikkim, Chandigarh, Delhi.
o Maharashtra leads in water conservation schemes.
o 78% are man-made water bodies whereas 22% are natural water bodies.
o 55.2% are owned by private entities whereas rest are under public ownership.
o Water User Associations (WUA) has helped in preventing encroachments.

Current Affairs- Easy


11. UNESCO World Press Freedom Prize 2023 has been awarded to Niloofar Hamedi, Elaheh Mohammadi and
Narges Mohammadi from-
(a) Afghanistan
(b) Sudan
(c) Pakistan
(d) Iran

ANSWER: D
• Three imprisoned women journalists- Niloofar Hamedi, Elaheh Mohammadi and Narges Mohammadi from
Iran have been awarded the UNESCO World Press Freedom Prize 2023 for their commitment to truth and
accountability.
o World Press Freedom Day is observed on 3rd May, every year.
• The UNESCO/Guillermo Cano World Press Freedom Prize was established in 1997.
• It is presented annually to a person, organization or institution that has made an outstanding contribution
to press freedom, and especially when this has been achieved in the face of danger.

Current Affairs- Medium


12. Consider the following statements:
1. Pradhan Mantri Urja Ganga Project aims at providing natural gas in western India.
2. The project is being implemented by GAIL.
Which of the following statements is/ are correct?
(a) 1 only
(b) 2 only
(c) Both 1 and 2
(d) Neither 1 nor 2

ANSWER: B
• PM recently praised the completion of Bihar portion of Barauni-Guwahati pipeline under Pradhan Mantri
Urja Ganga Project.
About Pradhan Mantri Urja Ganga Project
• The project is also known as Jagdishpur – Haldia & Bokaro – Dhamra Pipeline Project (JHBDPL) project.
• It was launched in 2016 and will cater to the energy requirements of five states namely Uttar Pradesh, Bihar,
Jharkhand, Odisha and West Bengal.
www.civilstaphimachal.com Contact: 7814622609

• Total length of pipeline under Pradhan Mantri Urja Ganga Project is 3,384 km, out of which 766 km of
pipeline is in Odisha state and the balance 2,618 km is in the states of Uttar Pradesh, Bihar, Jharkhand, West
Bengal & Assam.
• This pipeline will connect six districts in Bihar to the National Gas Grid.
• The project will provide access to clean natural gas for industries, CNG for vehicles and PNG for homes,
promoting a sustainable lifestyle.
• The seven main station cities include Varanasi, Patna, Bokaro, Jamshedpur, Kolkata, Ranchi, Bhubaneswar
and Cuttack as the major beneficiaries of the project.
• The main trunk of the pipeline ends at Haldia (West Bengal) and Dhamra (Odisha).
• The project is being implemented by GAIL.

Current Affairs- Medium


13. Consider the following statements:
1. World Intellectual Property Day is observed annually on April 28th.
2. Its theme this year is “Women and IP: Accelerating Innovation and Creativity.”
Which of the following statements is/ are correct?
(a) 1 only
(b) 2 only
(c) Both 1 and 2
(d) Neither 1 nor 2

ANSWER: B
• World Intellectual Property Day is observed annually on April 26th.
• The World Intellectual Property Organization (WIPO) officially established World IP Day on August 9, 1999,
coinciding with WIPO's 30th founding anniversary.
• April 26th was designated as the date for the annual observance of World Intellectual Property Day.
• The inaugural celebration of World IP Day took place in the year 2000, with 59 member countries
participating in the event.
• The primary objective of observing World Intellectual Property Day is to create consciousness about the
significance of protecting intellectual property.
• It serves as a platform to emphasize the importance of intellectual property protection and to generate
awareness about copyrights, patents, trademarks, and other related subjects.
• The theme for the World Intellectual Property Day 2023 is “Women and IP: Accelerating Innovation and
Creativity.”

Current Affairs- Medium


14. Consider the following statements:
1. Maria Stepanova was recently awarded the Leipzig Book Prize for European Understanding in 2023.
2. She won the award for her poetry ‘Girls Without Clothes’ about hidden violence against the female body
and the power imbalance.
Which of the following statements is/ are correct?
(a) 1 only
(b) 2 only
(c) Both 1 and 2
(d) Neither 1 nor 2

ANSWER: C
• Maria Stepanova, a renowned Russian writer currently residing in Berlin, has been awarded the Leipzig Book
Prize for European Understanding in 2023.
• It was her volume of poetry, Girls Without Clothes, that earned her the prestigious Leipzig Book Prize.
• Stepanova, a Russian-Jewish poet, novelist, and journalist born in Moscow in 1972, has been a vocal critic
of Vladimir Putin’s administration.
www.civilstaphimachal.com Contact: 7814622609

About the Leipzig Book Prize


• The Leipzig Book Prize for European Understanding is a prestigious literary award that has been awarded
annually since 1994.
• The prize aims to promote a greater understanding between European cultures by recognizing outstanding
works of fiction, non-fiction, and poetry that contribute to this goal.
• The prize is awarded as part of the Leipzig Book Fair in Germany and is presented in three categories: fiction,
non-fiction, and translation.
• The winner in each category receives a cash prize and a bronze sculpture.

Current Affairs- Difficult


15. Consider the following statements:
1. The Sundargarh natural arch is located in Chhattisgarh district.
2. Food Outlook report was recently released by FAO.
3. Pharmaceuticals & Medical Devices Bureau of India is implementing agency for Pradhan Mantri Bharatiya
Jan Aushadhi Pariyojana (PMBJP) scheme.
Which of the following statements is/are correct?
(a) 1 & 2 only
(b) 2 & 3 only
(c) 3 only
(d) 1, 2 and 3

ANSWER: B
• The Odisha state unit of the Geological Survey of India (GSI) has proposed to declare the ‘Natural Arch’ in
the Kanika range of Sundargarh forest division, a Geo Heritage Site.
o If it is done, it will be the biggest natural arch of the country to have the Geo Heritage tag.
• The Sundargarh natural arch, present in the ferruginous sandstone of the Upper Kamthi formation, dates
back to about 184 to 160 million years in the lower to middle Jurassic age.
• It is an oval-shaped arch and has a length of 30 metres at the base and is 12 metres high.
• FAO released Food outlook that contained forecasts of production, trade, utilisation and stock levels across
the world’s major basic foodstuffs.
o It is a biannual report.
o It estimates that the global food bill will rise to $1.98 trillion in 2023, up 1.5 percent from 2022. It rose
by 11 percent in 2022 and 18 percent in 2021.
o The import bill for the group of Least Developed Countries (LDCs) is predicted to decline by 1.5 % this
year and that for net food-importing developing countries (NFIDCs) to decline by 4.9%.
• PMBJP was launched by Department of Pharmaceuticals in 2008 with objective of making quality generic
medicines available at affordable prices to people.
• Nodal Ministry- Ministry of Chemicals and Fertilizers.
• Pharmaceuticals & Medical Devices Bureau of India (PMBI), erstwhile Bureau of Pharma PSUs of India (BPPI)
is implementing agency of PMBJP.

HPGK- Medium
16. Mark the correct answer :
1. Hathi Dhar is an inner ridge of the Dhauladhar ranges.
2. Pangi Range is an extension of the Dhauladhar ranges in Chamba.
3. Dagani Dhar is the smallest offshoot of the Bara Bhangal.

(a) Only statement 1 is correct.


(b) Only statements 2 and 3 are correct.
(c) Only statements 1 and 3 are correct.
(d) All statements are incorrect.

ANSWER: D
www.civilstaphimachal.com Contact: 7814622609

● Hathi Dhar, the inner ridge of the Shiwalik range, runs parallel to the southern flank of the Dhauladhar
range.
● Pangi Range, an extension of the magnificent Greater Himalayas in district of Chamba.
● Dagani Dhar is an offshoot of Pangi range .

HPGK- Tough
17. Select the incorrect pair:
Valleys District
(a) Jyuni : Mandi
(b) Sojha : Kullu
(c) Hudan : Sirmour
(d) Hassan : Shimla

ANSWER: C
● Some prominent valleys of Himachal Pradesh:
Valleys District
A. Jyuni : Mandi
B. Sojha : Kullu
C. Hudan : Chamba
D. Hassan : Shimla

HPGK- Tough
18. Select the incorrect statement/s:
1. The course of Beas river is longest in Himachal Pradesh compared to other rivers in Himachal Pradesh.
2. The catchment area of Satluj is higargest in Himachal Pradesh compared to other rivers of Himachal Pradesh.
3. Hydroatis,Hyphasis,and Hesidros were the Greek names of Ravi,Beas and Satluj respectively.
4. Yamuna has a mythical relation with Rishi Vashist.

(a) 1 and 2
(b) 2 and 3
(c) 3 and 4
(d) 1 and 4

ANSWER: D
● The course of Satluj river is longest in Himachal Pradesh compared to other rivers in Himachal Pradesh.(
320km)
● The catchment area of Satluj is largest in Himachal Pradesh compared to other rivers of Himachal Pradesh.
(20,000 sq. Km. )
● Hydroatis,Hyphasis,and Hesidros were the Greek names of Ravi,Beas and Satluj respectively.
● Beas has a mythical relation with Rishi Vashist not Yamuna river .

HPGK- Tough
19. Consider the following statements and choose correct statement:
1. Swanglas are endogamous .
2. Hypergamous marriage is a well recognised form of marriage among Swangla tribes .
3. Children of the Hypergamous union are called Garu and Mundro .
4. Both Junior levirate and Sororate are accepted customs of Swangla Tribe.

(a) 1 and 2 .
(b) 1,3 and 4
(c) 1,2 and 3
(d) 2,3 and 4
www.civilstaphimachal.com Contact: 7814622609

ANSWER: B
● Swanglas are endogamous(custom of marrying within one's own group or community). Marriage outside
one's own social order is not recognised, and all such persons are declared outcast .
● Hypergamous marriage is a not recognised form of marriage among Swangla tribes.
That Is why children of the Hypergamous union are called Garu and Mundro.
● Both Junior levirate and Sororate are accepted customs of Swangla Tribe.
● Junior Levirate:
○ The junior levirate is a custom in which a widow is expected to marry one of her deceased husband's
brothers, specifically the younger brother.
● Sororate:
○ The sororate is a custom in which a widower is expected to marry one of his deceased wife's sisters,
specifically the younger sister.

HPGK- Tough
20. Which of the following statements are true ?
1. Dr. Moti Chandra said about the Vishnu idol in Hari Rai temple, "No idol of this kind is known to exist."
2. The Lakshami Narain temple in Chamba is one of a group of six stone temples in a row .
3. Vajreshwari Devi and Indreshwar temples in Kangra town are built in Shikhara style .

(a) 1 only
(b) 1 and 2 only
(c) 1 and 3 only
(d) All of the above

ANSWER: D
● Dr. Moti Chandra said about the Vishnu idol in Hari Rai temple, "No idol of this kind is known to exist."
○ Hari Rai temple in Chamba has the ninth century image of Vishnu executed in the Chamba style. It
is a massive ashtadhatu idol.
● The Lakshami Narain temple in Chamba is one of a group of six stone temples in a row .Three of these are
dedicated to Vishnu and three to Mahadeva .The principal temple is dedicated to Narayana.
● Vajreshwari Devi and Indreshwar temples in Kangra town are built in Shikhara style .

HPGK- Medium
21. Match list-1 with list-2 and choose the correct statement:
Gompa Person
1. Kardang a. Padmasambhava
2. Sashur b. Deva Gyatso
3. Tayul c. Lama Norbu
4. Guru Ghantal d. Lama Sarja Rin Chain

(a) 1-a,2-b,3-c,4-d
(b) 1-c,2-d,3-b,4-a
(c) 1-d,2-c,3-a,4-b
(d) 1-c,2-b,3-d,4-a

ANSWER: D
● Lama Norbu renovated Kardang Gompa in 1912.
● Sashur Gompa was built in the 7th century by Deva Gyatso.
● Tayul Gompa was built in the 17th century by Lama Sarja Rin Chain.
● Guru Ghantal Gompa has been consecrated by Guru Padamsambhava.

HPGK (Survey) - Easy


www.civilstaphimachal.com Contact: 7814622609

22. Which sector in Himachal Pradesh witnessed the highest contraction during the fiscal year 2020-21 in GVA
terms ?
(a) Mining and quarrying sector
(b) Trade, hotels, and transport sector
(c) Agriculture and allied sectors
(d) Forestry and logging sector

ANSWER: B
● The sector that experienced the highest contraction during the fiscal year 2020-21 was the "Trade, hotels,
and transport sector."
● It had a contraction rate of 20.2%.
● It was primarily because of Covid .

HPGK- Medium
23. Match the following:
1. Himachal Pradesh Administrative Tribunal a.1971
2. Himachal Pradesh Judicial Academy b.1986
3. Himachal Pradesh High Court c. 2005
4. Himachal Pradesh Institute of Public Administration d.1974

(a) 1-a,2-b,3-c,4-d
(b) 1-c,2-d,3-b,4-a
(c) 1-b,2-c,3-a,4-d
(d) 1-c,2-b,3-d,4-a

ANSWER: C
● HP Administrative Tribunal:
○ Established in September 1986.
● HP Judicial Academy:
○ Established in November 2005.
● HP High Court:
○ In 1971, HP High Court was established with 1 Chief Justice and 10 judges .
● HIPA( Himachal Pradesh institute of Public Administration) :
○ Established on 25 April, 1971.

HPGK- Medium
24. Consider the following statements and choose correct answer:
1. The only temple of Manu in India lies in Mandi .
2. Jamlu temple is related to Rishi Jamadagni .
3. Hidimba Devi temple is a four tiered pagoda temple .

(a) 1 only
(b) 2 and 3 only
(c) 1 and 3 only
(d) 1,2, and 3

ANSWER: C
● The only temple of Manu in India lies in Kullu( Shanshar).
● Jamlu temple is situated in Malana and is related to Rishi Jamadagni known as Jamlu Devta .
○ He was the husband of Renuka Ji and father of Parshuram .
● Hidimba Devi temple is a four tiered pagoda temple.
○ It was built by Raja Bahadur Singh of Kullu in 1553 AD .
www.civilstaphimachal.com Contact: 7814622609

HPGK- Medium
25. Which of the following statements are true ?
1. Kalka -Shimla was included in UNESCO's World Heritage Site in 2008.
2. Initially, the conception of the Pathankot-Joginder Nagar railway line focused primarily on its role as a goods
carrier.
3. Longest narrow gauge bridge of India lies in Himachal Pradesh.

(a) 1 only
(b) 1 ,2 and 3
(c) 2 and 3
(d) 4 only

ANSWER: B
● The Kalka–Shimla Railway is a 2 ft 6 in (762 mm) narrow-gauge railway in North India which traverses a
mostly mountainous route from Kalka to Shimla.
○ On 8 July 2008, UNESCO added the Kalka–Shimla Railway to the mountain railways of India World
Heritage Site.
● Initially, the conception of the Pathankot-Joginder Nagar railway line focused primarily on its role as a goods
carrier.It was built by Britishers for transporting raw material to the Shanan Power House at Joginder Nagar
In Mandi .
● The Gaj Bridge(over Gaj Khadd) holds the record for the longest narrow gauge bridge in India.

HPGK- Easy
26. Tegpo and Kabzian are tributaries of which river :
(a) Soldang
(b) Spiti
(c) Satluj
(d) Throng

ANSWER: B
● Tegpo and Kabzian are tributaries of Spiti river .
● Lipak, Tirasang, Chaladogpo and Yulang are other tributaries of Spiti river.

HPGK- Medium
27. Consider the statements and choose correct answer:
1. First IT park of Himachal Pradesh was established at Mauja Majhol.
2. Paragpur was the first village In India to be given Heritage Status .
3. First Governor -General to visit Himachal Pradesh was Lord Curzon.

(a) 1 only
(b) 2 only
(c) 1 and 2 only
(d) 1,2 and 3

ANSWER: D
● First IT park of Himachal Pradesh was established at Mauja Majhol( Waknaghat,Solan).
● Paragpur(Kangra) was the first village In India to be given Heritage Status in 1997.
● First Governor -General to visit Himachal Pradesh was Lord Amherst( In 1827 A.D).

HPGK- Medium
28. Which of the following pairs is incorrect ?
1. Narbadeshwar : Flat Roofed
www.civilstaphimachal.com Contact: 7814622609

2. Chintpurni : Domed
3. Adi Brahma : Pent -roof
4. Bijali Mahadev : Pagoda

(a) 1 and 2
(b) 3 and 4
(c) 1 and 4
(d) 2 and 3

ANSWER: B
● Narbadeshwar( Sujanpur) is the flat-roofed temple of Kangra valley.
● Chintpurni( Una) temple is built in Dome style.
● Adi Brahma temple ( Kullu) is built in Pagoda style.
● Bijali Mahadev is built in Pent-Roof style .

HPGK- Medium
29. In Himachal Pradesh, which monastery holds the distinction of being considered the second most sacred
monastery after the Tholang Monastery in Sikkim?
(a) Tabo
(b) Kee
(c) Tayul
(d) Gemur

ANSWER: A
● Tabo Monastery holds the distinction of being considered the second most sacred after the Tholang
Monastery in Sikkim.
● Tholung Monastery is a gompa located in the buffer zone of Khangchendzonga National Park. It is
considered one of the most sacred monasteries in Sikkim.

HPGK(Survey) - Medium
30. Mukhya Mantri Krishi Utpadan Sanrakshan Yojana (MMKUSY) is an umbrella scheme comprising of which
of the following sub schemes:
1. Solar fencing scheme
2. Flow irrigation scheme
3. Anti hail net structure
4. Green House Renovation scheme

(a) 1 and 2
(b) 1,2 and 3
(c) 1,3 and 4
(d) 2,3 and 4

ANSWER: C
● The three protection-based Departmental schemes that were already in place and had the same goal were
combined into a single scheme called "Mukhya Mantri Krishi Upadan Sanrakshan Yojana," which has three
parts.
○ Sour Badhbandi (Solar Fencing)
○ Anti Hail-Net Structure
○ Green House Renovation Scheme

HPGK(Survey) - Tough
31. Consider the following statements and choose the correct answer:
1. As per the Livestock Census 2019, the State has 0.82 per cent share of India’s total livestock.
www.civilstaphimachal.com Contact: 7814622609

2. The total livestock population in the State stood at 44.13 lakh according to Livestock Census 2019.
3. The State ranks 20th in Cattle population in the country according to Livestock Census 2019.

(a) 1 only
(b) 2 and 3
(c) 1,2 and 3
(d) None

ANSWER: C
● As per the Livestock Census 2019, the State has 0.82 per cent share of India’s total livestock and 0.16 per
cent of the total poultry.
● The total livestock population in the State stood at 44.13 lakh, and that of the poultry population was 13.42
lakh.( Livestock Census 2019).
● The State ranks 20th in Cattle and 27th in Poultry population in the country.( Livestock Census 2019).

HPGK- Tough
32. Select the incorrect pair :
1. Forest cover : 37,948 sq km
2. Very dense forest : 3,163 sq km
3. Moderate dense forest : 7,100 sq km
4. Open Forest : 5,180 sq km

(a) 1 only
(b) 2 only
(c) 3 only
(d) 4 only

ANSWER: A
● Forest types of Himachal Pradesh:
Forest cover : 15,443 sq km ( 27.74 %)
Forest Area : 37,948 sq km (68.16 % )
Very Dense forest : 3,163 sq km
Moderate dense forest : 7,100 sq km
Open Forest : 5,180 sq km

HPGK(Survey) - Tough
33. Consider the following statements and choose correct answer:
1. The Department of Environment, Science & Technology is implementing the Model Eco Villages scheme in
the State.
2. The State Government has banned use and littering of plastic items from time to time under Himachal
Pradesh Non-Biodegradable Garbage (Control) Act, 2005.
3. The Vidyarthi Van Mittar yojna is being executed in partnership with the Education Department and in
conjunction with the Himachal Pradesh State Legal Services Authority.

(a) 1 and 2
(b) 2 and 3
(c) 3 and 4
(d) 1 and 3

ANSWER: D
● The Model Eco Villages scheme is being implemented by the Department of Environment, Science &
Technology in the State.
www.civilstaphimachal.com Contact: 7814622609

○ The primary goal of this scheme is to foster the development of a low impact lifestyle, aiming to
reduce the "ecological footprint" by up to 50 percent compared to the initial assessment made at
the scheme's inception.
● From time to time, the State Government has implemented a ban on the use and disposal of plastic items
in accordance with the Himachal Pradesh Non-Biodegradable Garbage (Control) Act of 1995.
○ This Act encompasses a complete prohibition on polythene bags, single-use plastic spoons, bowls,
katories, stirring sticks, forks, knives, and straws made of plastic and thermocol in Himachal Pradesh.
● In line with the National Mission for Sustaining Himalayan Ecosystems (NMSHE), the Department of
Environment, Science, and Technology in Himachal Pradesh has established the State Knowledge Cell on
Climate Change (KCCC).
○ The department is actively developing a comprehensive database that encompasses geological,
hydrological, ecological, socio-cultural, and traditional ecosystem preservation and conservation
information.
○ This data is closely monitored and analyzed to establish a foundation of knowledge for climate
change policies.
○ The Climate Change Vulnerability Assessment and Adaptation Plans have been prepared for the
districts of Kinnaur and Lahaul-Spiti, while similar plans for Shimla, Kullu, and Mandi are currently
under preparation.

HPGK(Survey) - Tough
34. Consider the following statements and choose correct answer:
1. A center for inspecting and certifying vehicle fitness has been established in Baddi, Himachal Pradesh.
2. Himachal Pradesh is not included in the list of the top 10 states preferred by domestic and international
tourists as tourist destinations.
3. Established in 1974, Himachal Pradesh Tourism Development Corporation has played a pioneering role in
the development of tourism infrastructure in Himachal Pradesh.
4. Nai Raahein Nai Manzilein scheme was launched in 2018-19 in Himachal pradesh.

(a) 1 ,2 and 4
(b) 2,3 and 4
(c) 1 and 4
(d) 1,2,3 and 4

ANSWER: A
● The Ministry of Road Transport and Highways (MoRTH) has approved a project to establish an Inspection
and Certification Centre for vehicle fitness in Baddi, Himachal Pradesh, with a project cost of ₹16.35 crore.
● The State does not figure among the top 10 tourist destination states for domestic as well as foreign tourist
arrivals, as per the statistics released by the Ministry of Tourism, Government of India in 2022.
● HPTDC(Himachal Pradesh Tourism Development Corporation) is a pioneer in the development of tourism
infrastructure in Himachal Pradesh formed in 1972.
○ It provides a complete package of tourism services, including accommodation catering, transport,
conferencing and sports activities, having the largest chain of finest hotels and restaurants in the
State.
● To develop new tourist destinations, the state Government has implemented a new scheme “Nai Rahein
Nai Manzilein” in the year 2018-19, to develop the unexplored areas of the state from a tourism point of
view.

HPGK) - Easy
35. Who are believed to be the descendants of Huns in Himachal Pradesh?
(a) Koli
(b) Hali
(c) Gujjars
(d) Rihara
www.civilstaphimachal.com Contact: 7814622609

ANSWER: C
● Gujjars in Himachal Pradesh are believed to be the descendants of Huns.

ECONOMY - Tough
36. Under which Five Year Plan, did agriculture register a negative growth rate in India ?
(a) First
(b) Fifth
(c) Fourth
(d) Third

ANSWER: D
● The Third Five-year Plan( 1961-66) stressed agriculture and improvement in the production of wheat, but
the brief Sino-Indian War of 1962 exposed weaknesses in the economy and shifted the focus towards the
defence industry and the Indian Army.
● In 1965–1966, India fought a War with Pakistan. There was also a severe drought in 1965.
● Agriculture demonstrated negative growth in the Third Five Year Plan.
● During the years 1961-65, the growth rate of agriculture was recorded at – 1.0%.
● Third five year plan was based on John Sandy and Sukhamoy Chakraborty's model.

ECONOMY - Tough
37. With reference to the National Food Security Act, which of the following statements
is/are correct?
1. It will cover upto 75 percent rural and 50 percent urban population.
2. Special focus on nutritional support to women and children.
3. Eldest woman above 18 years of age will be head of household.

Select the correct answer using the codes given below :


(a) 1 and 2 are correct
(b) 2 and 3 are correct
(c) 1, 2 and 3 are correct
(d) None of these

ANSWER: C
● The enactment of the National Food Security Act (NFSA), 2013 marks a paradigm shift in the approach to
food security from welfare to rights based approach. Major provisions of NFSA, 2013 are :
○ The Act legally entitled upto 75% of the rural population and 50% of the urban population to receive
subsidised foodgrains under Targeted Public Distribution System, thus covering about two-thirds of
the population.
○ The eligible households will be entitled to receive 5 kg food grains per person per month at Rs. 3,2,1
per kg for rice, wheat and coarse grains, respectively.
○ The existing Antyodaya Anna Yojana (AAY) households (i.e. poorest of the poor) will continue to
receive 35 kg of foodgrains per household per month.
○ The oldest woman in the household, 18 years or above, shall be head of the household for the
issuance of the ration card.
○ The Act also has a special focus on the nutritional support to women and children.

ECONOMY - Easy
38. Indian Model of Development ensures interest of:
(a) Person
(b) State
(c) State and Person both
(d) None of the above
www.civilstaphimachal.com Contact: 7814622609

ANSWER: C
● India is a mixed type of economy. Under the mixed economy both public and private sectors participate in
the development of the country.
● The administration of the public sector is state governed whereas the administration of the private sector
is in the hands of a person.
● Thus, the Indian model of development protects the interests of both the state and the individual.

ECONOMY -Medium
39. Consider the following pairs :
Organization Headquarters
1. Asian Development Bank : Tokyo
2. Asia-Pacific Economic Cooperation : Singapore
3. Association of South-East Asian Nations : Bangkok
4. Asian Infrastructure Investment Bank : Shanghai
Which of the above pair(s) is/are correctly matched?
(a) 1 and 2
(b) 2 only
(c) 2 and 3
(d) 3 only

ANSWER: B
● Asian Development Bank(ADB) : Manila, Philippines(HQ)
● Asia-Pacific Economic Cooperation (APEC) : Singapore(HQ)
● Association of South-East Asian Nations(ASEAN) : Jakarta, Indonesia(HQ)
● Asian Infrastructure Investment Bank (AIIB) : Beijing, China(HQ)

ECONOMY -Tough
40. The economist who for the first time scientifically determined National Income in India was :
(a) D.R. Gadgil
(b) V.K.R.V. Rao
(c) Dadabhai Nao roji
(d) Y.V. Alagh

ANSWER: B
● The first attempt to calculate National Income of India was made by Dadabhai Naoroji in 1867-68,
● While for the first time National Income of India was scientifically determined by Dr. V.K.R.V. Rao in 1931-
32.

ECONOMY -Tough
41. Consider the following statements about Non-Tax Revenue?
1. It is the recurring income earned by the government from sources and taxes together.
2. Though the Railways are a separate department, all their receipts and expenditure are routed through the
Consolidated Fund.
Which of the above given statement(s) is/are incorrect?
(a) Only 1
(b) Only 2
(c) Both 1 and 2
(d) Neither 1 nor 2

ANSWER: A
Statement 1 is incorrect.
Non-Tax Revenue is the recurring income earned by the government from
www.civilstaphimachal.com Contact: 7814622609

sources other than taxes.


Statement 2 is correct.
Though the Railways are a separate department, all their receipts and expenditure are routed through the
Consolidated Fund.

ECONOMY - Tough
42. Which of the following can be said to be essentially the parts of Inclusive Governance?
1. Allowing the Non-Banking Financial Companies to do banking
2. Establishing effective District Planning Committees in all the districts
3. Increasing the government spending on public health
4. Strengthening the Mid-day Meal Scheme
Select the correct answer using the codes given below :
(a) 1 and 2 only
(b) 3 and 4 only
(c) 2, 3 and 4 only
(d) 1, 2, 3 and 4

ANSWER: C
● Inclusive governance means that all sections of the society should be equally provided with the facilities
provided by the government.
● To establish effective District Planning Committees in all the districts, to increase government expenditure
on public health and to empower Mid-day Meal Scheme are all parts of inclusive governance.
● Permitting the Non-Banking Financial Companies to do banking can not be considered as part of inclusive
governance.

ECONOMY- Tough
43. Which of the following statements are true regarding “Shadow Banking”:
1. They by and large raise funds from the market
2. Their activities are transparent and properly regulated
3. These are mostly non banking financial institutions
Select the correct answer using the code given below:
(a) 1 only
(b) 2 only
(c) 3 only
(d) 1 and 3

ANSWER: D
● Shadow banking operates outside the regular banking system and their financial activities are undertaken
with less transparency and regulation than the conventional banking.
● While commercial banks, by and large, derive funds through mobilization of public deposits, shadow banks
raise funds, by and large, through market-based instruments such as commercial paper, debentures, or
other structured credit instruments.
● They are fundamentally distinct from commercial banks in various respects. NBFCs are examples of shadow
banks.

ECONOMY -Tough
44. Which of the following committees has given its recommendations on ‘Financial Inclusion’?
(a) Verma committee
(b) Pandey Committee
(c) Rangarajan Committee
(d) Kelkar Committee

ANSWER: C
www.civilstaphimachal.com Contact: 7814622609

● The Committee on Financial Inclusion was constituted by the Government of India (Chairman Dr. C.
Rangarajan) on June 26, 2006 to prepare a strategy of financial inclusion. The Committee submitted its final
Report on January 4, 2008.
● The Report viewed financial inclusion as a comprehensive and holistic process of ensuring access to financial
services and timely and adequate credit, particularly by vulnerable groups such as weaker sections and low
income groups at an affordable cost.
● Financial inclusion, therefore, according to the Committee, should include access to mainstream financial
products such as bank accounts, credit, remittances and payment services, financial advisory services and
insurance facilities.

ECONOMY -Medium
45. Which of the following schemes are related to skill development?
1. STRIDE
2. STRIVE
3. SANKALP
4. SHREYAS
Select the correct answer using the codes given below :
(a) 1 and 2 only
(b) 3 and 4 only
(c) 2, 3 and 4 only
(d) 1, 2, 3 and 4

ANSWER: C
● The 'Scheme for Trans-disciplinary Research for India's Developing Economy' (STRIDE) was launched by the
University Grants Commission (UGC). STRIDE aims to provide support to research projects that are socially
relevant, logically need-based, nationally important and globally significant. Hence, STRIDE is not a skill
development scheme.
● STRIVE (Skills Strengthening for Industrial Value Enhancement);
● SANKALP (Skill Acquisition and Knowledge Awareness for Livelihood Promotion);and
● SHREYAS (Scheme for Higher Education Youth in Apprenticeship and Skills) are related to skill development.

Polity -Easy
46. Under which act, a separate representation was introduced for Universities, Zamindars and Presidency
Corporations?
(a) Indian Council Act of 1892
(b) Indian Council Act of 1909
(c) Government of India Act of 1919
(d) Government of India Act of 1935

ANSWER: B
Indian Council Act of 1909 provided for the separate representation of presidency corporations, chambers of
commerce, universities and zamindars.

Polity -Easy
47. Which one of the following is not a provision related to a Money Bill?
(a) Imposition, abolition, remission, alteration or regulation of any tax.
(b) Appropriation of money out of Consolidated Fund of India.
(c) Imposition of fines by local authority for local purpose.
(d) Custody of the Consolidated Fund of India or the Contingency Fund of India.

ANSWER: C
A Money Bill is defined under Article 110 of the Indian Constitution and contains provisions that specifically
deal with matters related to taxation, appropriation of money from the Consolidated Fund of India, or the
www.civilstaphimachal.com Contact: 7814622609

custody of funds such as the Consolidated Fund of India or the Contingency Fund of India. The provision related
to the imposition of fines by a local authority for a local purpose is not considered a provision related to a
Money Bill because it does not fall within the specific financial matters outlined in Article 110 of the Indian
Constitution.

Polity- Medium
48. Consider the following statements about North-Eastern Council:
1. It was created by North-Eastern council Act of 1971.
2. Sikkim was added to the North-Eastern council in the year 2002.
Which of the following is/are correct?
(a) 1 only
(b) 2 only
(c) Both 1 and 2
(d) Neither 1 nor 2

ANSWER: C
Statement 1 is correct: North-Eastern Council was created by a separate Act of Parliament—the North-Eastern
Council Act of 1971. Its members include Assam, Manipur, Mizoram, Arunachal Pradesh, Nagaland, Meghalaya,
Tripura and Sikkim.
Statement 2 is correct: The State of Sikkim has been included in the North Eastern Council through North
Eastern Council (Amendment) Act, 2002 which was notified on 23rd December 2002.

Polity- Tough
49. Consider the following statements:
1. The Supreme Court upheld the constitutional validity of the National Awards- Bharat Ratna, Padma
Vibhushan etc. and these can be used as suffixes or prefixes to the names of awardees.
2. The Janata Party government had discontinued the National Awards- Bharat Ratna, Padma Vibhushan etc.
in 1977.
Which of the following is/are correct?
(a) 1 only
(b) 2 only
(c) Both 1 and 2
(d) Neither 1 nor 2

ANSWER: B
Statement 1 is incorrect: In 1996, the Supreme Court upheld the constitutional validity of the National Awards-
Bharat Ratna, Padma Vibhushan, Padma Bhushan and Padma Sri. It said that these awards do not amount to
‘titles’ within the meaning of Article 18 that prohibits only hereditary titles of nobility. They do not violate
Article 18 as the theory of equality does not mandate that merit should not be recognized. But they should
not be used as suffixes or prefixes to the names of awardees. Otherwise, the awardees should forfeit the
awards.
Statement 2 is correct: These National Awards were instituted in 1954. The Janata Party government headed
by Morarji Desai discontinued them in 1977. But they were again revived in 1980 by the Indira Gandhi
government.

Polity- Tough
50. Consider the following Statements:
1. The 6th Constitutional Amendment Act of 1956 facilitated the appointment of the same person as a
governor for two or more states.
2. The Chief justice of the concerned state high court may be appointed temporarily to discharge the functions
of the governor of that state.
Which of the following is/are correct?
(a) 1 only
www.civilstaphimachal.com Contact: 7814622609

(b) 2 only
(c) Both 1 and 2
(d) Neither 1 nor 2

ANSWER: B
Statement 1 is incorrect: The 7th Constitutional Amendment Act of 1956 facilitated the appointment of the
same person as a governor for two or more states.
Statement 2 is correct: Article 159 of the Constitution provides for the appointment of the Chief Justice of a
High Court to discharge the functions of the Governor of a state if the office of the Governor falls vacant or if
the Governor is unable to discharge his or her functions. This can happen due to various reasons such as illness,
absence, resignation, removal, or death of the Governor. In such a case, the Chief Justice of the concerned
state High Court may be appointed temporarily to discharge the functions of the Governor until a new
Governor is appointed or until the Governor is able to resume his or her duties. This provision ensures that the
governance of the state is not affected in case of a vacancy or incapacity of the Governor.

Polity- Medium
51. Given below are two statements:
Statement I: Lok Adalat is based on Gandhian principles and has been given statutory status under Legal
Services Authorities Act, 1986.
Statement II: An award passed by Lok Adalat is final and no appeal is maintainable from it.
In light of the above statements, choose the most appropriate answer from the options given below:
(a) Both Statement I and Statement II are correct.
(b) Both Statement I and Statement II are incorrect.
(c) Statement I is correct and Statement II are incorrect.
(d) Statement I is incorrect and Statement II is correct.

ANSWER: D
Statement I is incorrect: Lok Adalat is based on Gandhian principles and has been given statutory status under
Legal Services Authorities Act, 1987.
Statement II is correct: The Award passed by the Lok Adalat is deemed to be decree of a civil court. An award
passed by Lok Adalat is final and no appeal is maintainable from it.

Polity- Medium
52. Match the following:

(Articles) (Provisions)
a). Art. 16 1. No person shall be deprived of his property save by the authority of law.
b). Art. 29 2. No person can be discriminated against in the matter of public appointment on the ground
of race, religion, or caste.
c). Art. 30 3. All minorities whether based on religion or language shall have the Fundamental Rights to
establish and administer educational institution of their choice.
d). Art. 31 4. No citizen shall be denied admission into any educational institution maintained by the
state on receiving state aid, on grounds of religion, race, caste, language or any of them.

(a) a-2 b-4 c-3 d-1


(b) a-3 b-1 c-2 d-4
(c) a-2 b-1 c-3 d-4
(d) a-3 b-4 c-2 d-1

ANSWER: A
a)—2, Art. 16 -- No person can be discriminated against in the matter of public appointment on the ground of
race, religion, or caste.
www.civilstaphimachal.com Contact: 7814622609

b)—4, Art. 29 --No citizen shall be denied admission into any educational institution maintained by the state
on receiving state aid, on grounds of religion, race, caste, language or any of them.
c)—3, Art. 30 -- All minorities whether based on religion or language shall have the Fundamental Rights to
establish and administer educational institution of their choice.
d)—1, Art. 31 -- No person shall be deprived of his property save by the authority of law

Polity- Medium
53. Which of the following Acts are associated with the communal representation?
1. Indian Council Act, 1909
2. Government of India Act, 1919
3. Government of India Act, 1935
4. Rowlatt Act, 1919
Codes:
(a) 1 and 2 only
(b) 3 and 4 only
(c) 1, 2 and 3 only
(d) 1, 2, 3 and 4

ANSWER: C
Statement 1 is correct: The Indian Council Act, 1909 introduced a system of communal representation for
Muslims by accepting the concept of ‘separate electorate’. Under this, the Muslim members were to be elected
only by Muslim voters. Thus, the Act ‘legalised communalism’ and Lord Minto came to be known as the Father
of Communal Electorate
Statement 2 is correct: The Government of India Act, 1919 extended the principle of communal representation
by providing separate electorates for Sikhs, Indian Christians, Anglo-Indians and Europeans.
Statement 3 is correct: The Government of India Act, 1935 further extended the principle of communal
representation by providing separate electorates for depressed classes (scheduled castes), women and labour
(workers).
Statement 4 is incorrect: The British government enacting the Anarchical and Revolutionary Crimes Act of
1919, popularly known as the Rowlatt Act. It gave the government enormous powers to repress political
activities and allowed detention of political prisoners without trial for two years. It was not associated with
the communal representation.

Polity- Easy
54. Consider the incorrect statement/s:
1. 5th schedule contains the provisions for allocation of seats in the Rajya Sabha to the Union Territories.
2. 12th schedule was added by the 73rd Amendment Act, 1992.
3. 6th schedule contains the provisions for administration of tribal areas in Mizoram.
Codes:
(a) 2 only
(b) 1 & 2 only
(c) 2 & 3 only
(d) 1, 2 and 3

ANSWER: B
Statement 1 is incorrect: Fourth Schedule contains the provisions of allocation of seats in the Rajya Sabha to
the states and the union territories.
Statement 2 is incorrect: Eleventh Schedule (Article 243-G) specifies the powers, authority and responsibilities
of Panchayats. It has 29 matters. This schedule was added by the 73rd Amendment Act of 1992.
Statement 3 is correct: Sixth Schedule contains the provisions relating to the administration of tribal areas in
the states of Assam, Meghalaya, Tripura and Mizoram.

Polity- Tough
www.civilstaphimachal.com Contact: 7814622609

55. The High Court enjoys the power:


1. To issue writs for the enforcement of rights conferred on the citizens of India.
2. To exercise superintendence over the working of Courts and Tribunals under its jurisdiction.
3. To make general rules and prescribe forms regulating the practices and proceedings of Courts under its
jurisdiction.
Which of the following is/are correct?
(a) 1 only
(b) 1 and 3 only
(c) 2 and 3 only
(d) 1, 2 and 3

ANSWER: D
Statement 1 is correct: The High Court has the power to issue writs for the enforcement of fundamental rights
conferred on the citizens of India. These writs include habeas corpus, mandamus, prohibition, certiorari, and
quo warranto. The High Court uses this power to safeguard the fundamental rights of individuals and ensure
their protection.
Statement 2 is correct: The High Court exercises superintendence over the working of subordinate courts and
tribunals within its jurisdiction. This means that the High Court has the authority to oversee and supervise the
functioning of lower courts and tribunals to ensure that they are functioning properly and adhering to the law.
Statement 3 is correct: The High Court has the power to make general rules and prescribe forms that regulate
the practices and proceedings of the courts under its jurisdiction. These rules and forms help establish
procedural guidelines and ensure uniformity in the functioning of the courts within the High Court's
jurisdiction.

Polity- Tough
56. Consider the following functionaries:
1. Cabinet Secretary
2. Chief Election Commissioner
3. Union Cabinet Ministers
4. Chief Justice of India.
Their correct sequence, in the order of Precedence is:
(a) 3, 4, 2, 1
(b) 4, 3, 1, 2
(c) 4, 3, 2, 1
(d) 3, 4, I, 2

ANSWER: C
The correct sequence, in the order of precedence, is as follows:
1. Chief Justice of India: The Chief Justice of India holds the highest position in the Indian judiciary and is
considered the head of the Indian judiciary system. Therefore, the Chief Justice of India has the highest
precedence among the functionaries listed.
2. Union Cabinet Ministers: Union Cabinet Ministers are members of the executive branch of the
government and hold important ministerial positions in the central government. They have the second-
highest precedence after the Chief Justice of India.
3. Chief Election Commissioner: The Chief Election Commissioner is the head of the Election Commission
of India, which is responsible for conducting elections in the country. The Chief Election Commissioner
holds an important constitutional position and has precedence over the Cabinet Secretary.
4. Cabinet Secretary: The Cabinet Secretary is the highest-ranking civil servant in the government of India.
While the Cabinet Secretary holds a significant position, the functionaries listed above (Chief Justice of
India, Union Cabinet Ministers, Chief Election Commissioner) have higher precedence.
Therefore, the correct sequence in the order of precedence is:
1. Chief Justice of India
2. Union Cabinet Ministers
www.civilstaphimachal.com Contact: 7814622609

3. Chief Election Commissioner


4. Cabinet Secretary

Polity- Tough
57. The Constitution empowers the Parliament to make laws on any matter enumerated in the State List. In
light of the above statement, which of the following Laws were passed by the Parliament?
1. Prize Competition Act,1955
2. Wild Life (Protection)Act, 1972
3. Urban Land (Ceiling and Regulation) Act, 1976
4. Transplantation of Human Organs Act,1994
5. Geneva Convention Act, 1960
Codes:
(a) 1, 2, 3 and 5 only
(b) 1, 2, 4 and 5 only
(c) 2, 3, 4 and 5 only
(d) 1, 2,3 ,4 and 5

ANSWER: D
Statement 1 is correct: Prize Competition Act, 1955: This law have been passed under the provision
mentioned, as it relates to the regulation of prize competitions, which falls under the State List.
Statement 2 is correct: Wild Life (Protection) Act, 1972: This law been passed under the provision mentioned
because wildlife and conservation fall under the Concurrent List (42nd amendment act shifted this subject
from state list to concurrent list), where both the Parliament and State Legislatures have the authority to make
laws. The Wildlife (Protection) Act, 1972, was passed by the Parliament under this concurrent jurisdiction.
Statement 3 is correct: Urban Land (Ceiling and Regulation) Act, 1976: This law passed under the provision
mentioned as it pertains to urban land and its regulation, which falls under the State List.
Statement 4 is correct: Transplantation of Human Organs Act, 1994: This law passed under the provision
mentioned as it deals with the transplantation of human organs, which falls under the State List.
Statement 5 is correct: Geneva Convention Act, 1960: This law passed under the provision mentioned, as it
relates to the implementation of the Geneva Conventions in India, which falls under the State List.

Geography- Medium
58. Consider the following statements regarding Black soils
1) The black color is due to the presence of a small proportion of titaniferous magnetite of the parent rock.
2) The black soils are generally clayey, deep and impermeable.
Which of the statements given above is/are correct?
(a) 1 only
(b) 2 only
(c) Both 1 and 2
(d) Neither 1 nor 2

ANSWER: C
Black soils (Vertisols)
• The black color is due to the presence of a small proportion of titaniferous magnetite or iron and black
constituents of the parent rock.
• Black soils are very dark and have a very high clay content. They have a high moisture retention capacity.
They become extremely hard on drying and sticky on wetting. Hence, they are very difficult to cultivate and
manage.
• These soils are also known as the ‘Regur Soil’ or the ‘Black Cotton Soil’.
• The black soils are generally clayey, deep and impermeable.
• These soils cover an area of about 74 million ha, mainly in the central, western and southern states of India.
They are inherently very fertile.
• Under rainfed conditions, they are used for growing cotton, millets, soybean, sorghum, pigeon pea, etc.
www.civilstaphimachal.com Contact: 7814622609

Geography- Difficult
59. Consider the following statements regarding Kashmir Himalayas
1) The cold desert lies between the Greater Himalayas and the Karakoram ranges.
2) Between the Great Himalayas and the Pir Panjal range, lies the world-famous valley of Kashmir and the
famous Dal Lake.
3) The Kashmir Himalayas are famous for Karewa formations.
Which of the statements given above are correct?
(a) 1 and 2 only
(b) 2 and 3 only
(c) 1 and 3 only
(d) 1, 2 and 3

ANSWER: D
Kashmir Himalayas
• It comprises a series of ranges such as the Karakoram, Ladakh, Zaskar and Pir Panjal.
• The northeastern part of the Kashmir Himalayas is a cold desert, which lies between the Greater Himalayas
and the Karakoram ranges.
• Between the Great Himalayas and the Pir Panjal range, lies the world famous valley of Kashmir and the
famous Dal Lake.
• Important glaciers of South Asia such as the Baltoro and Siachen are also found in this region.
• The Kashmir Himalayas are also famous for Karewa formations, which are useful for the cultivation of
Zafran, a local variety of saffron.
• Some of the important passes of the region are Zoji La on the Great Himalayas, Banihal on the Pir Panjal,
Photu La on the Zaskar and Khardung La on the Ladakh range. Some of the important fresh lakes such as
Dal and Wular and salt water lakes such as Pangong Tso and Tso Moriri are also in this region.

Geography- Medium
60. Consider the following statements:
Statement I: While a lunar eclipse can occur twice a year, a solar eclipse occurs only once a year.
Statement II: A solar eclipse always takes place on the summer solstice of the Northern Hemisphere.
In the light of the above statements choose the most appropriate answer from the answer given below:
(a) Both statements I and statement II are correct.
(b) Both statements I and statement II are incorrect.
(c) Statement I is correct and statement II is incorrect.
(d) Statement I is incorrect and statement II is correct.

ANSWER: B
• A solar eclipse occurs when the Moon passes between Earth and the Sun, thereby obscuring the view of
the Sun from a small part of the Earth, totally or partially. Such an alignment occurs approximately every
six months, during the eclipse season in its new moon phase, when the Moon's orbital plane is closest to
the plane of the Earth's orbit.
• If the Moon were in a perfectly circular orbit and in the same orbital plane as Earth, there would be total
solar eclipses once a month, at every new moon. Instead, because the Moon's orbit is tilted at about 5
degrees to Earth's orbit, its shadow usually misses Earth. Solar (and lunar) eclipses therefore happen only
during eclipse seasons, resulting in at least two, and up to five, solar eclipses each year, no more than two
of which can be total.
• The summer solstice is the longest day of the year. The summer solstice occurs when Earth, orbiting the
sun, reaches its maximum tilt towards the sun. Earth experiences two solstices in a year -- summer and
winter solstice with the latter occurring when the Earth is tilting the farthest away from the sun. Summer
solstice solar eclipse is a rare phenomenon. The last time this occurred was in the year 2001 -- 19 years ago
www.civilstaphimachal.com Contact: 7814622609

that too on June 21, 2001, and before that, it occurred in the year 1982 -- June 21, 1982. And the next time
it is expected to occur is in the year 2039 on -- June 21. So, it is a rare phenomenon.
• With moon cycling through its phases every 29.53 days, 3.4 percent of all solstices will have a new Moon
phase in 12 hours of the moment of maximum tilt. Since we have two solstices, the probability only rises
to 13.5 percent, making it not very rare for a moon to coincide with a solstice.

Geography – Easy
61. Which among the following statement is incorrect about the peninsular plateaus?
(a) The general elevation of Peninsular plateau is from the east to the west.
(b) Delhi Ridge is the northwest extension of the peninsular plateau.
(c) The north-eastern extension of peninsular plateau is separated by Malda fault.
(d) Ravines of Madhya Pradesh are relief features of Peninsular plateau.

ANSWER: A
• General elevation of peninsular plateau is from west to east. That is why most rivers in peninsular India
flow towards Bay of Bengal. Hence , statement a is incorrect.
• Karbi Anglong and Shillong are North eastern Extension of peninsular plateau .
• The Ravines of Chambal, Bhind and Morena are a part of Peninsular plateau.

Geography- Difficult
62. Consider the following pairs :
Type of Rural Settlement Associated region
1) Clustered Settlement : Meghalaya region
2) Hamleted Settlement : Lower Ganga plain
3) Dispersed Settlement : Bundelkhand region

Which of the pairs given above is/are correctly matched ?


(a) 1 and 2 only
(b) 1 and 3 only
(c) 2 only
(d) 1, 2 and 3

ANSWER: C
Rural settlements in India can broadly be put into four types:
• Clustered Settlements is a compact or closely built up area of houses. In this type of village the general living
area is distinct and separated from the surrounding farms, barns and pastures. Such settlements are
generally found in fertile alluvial plains and in the northeastern states. Sometimes, people live in compact
village for security or defence reasons, such as in the Bundelkhand region of central India and in Nagaland.
In Rajasthan, scarcity of water has necessitated compact settlement for maximum utilisation of available
water resources.
• Semi-clustered or fragmented settlements may result from tendency of clustering in a restricted area of
dispersed settlement. More often such a pattern may also result from segregation or fragmentation of a
large compact village. Such settlements are widespread in the Gujarat plain and some parts of Rajasthan.
• Hamleted Settlements is fragmented into several units physically separated from each other bearing a
common name. These units are locally called panna, para, palli, nagla, dhani, etc. in various parts of the
country. This segmentation of a large village is often motivated by social and ethnic factors. Such villages
are more frequently found in the middle and lower Ganga plain, Chhattisgarh and lower valleys of the
Himalayas.
• Dispersed or isolated settlement pattern in India appears in the form of isolated huts or hamlets of few huts
in remote jungles, or on small hills with farms or pasture on the slopes. Extreme dispersion of settlement is
often caused by extremely fragmented nature of the terrain and land resource base of habitable areas.
Many areas of Meghalaya, Uttaranchal, Himachal Pradesh and Kerala have this type of settlement.
www.civilstaphimachal.com Contact: 7814622609

Geography- Difficult
63. With reference to the occurrence of minerals across India, consider the following pairs:
Mineral Mine
1) Copper : Khetri
2) Aluminum : Balaghat
3) Iron Ore : Gurumahisani
4) Manganese : Panchpatmali Hills
Which of the pairs given above are correctly matched?
(a) 1, 2 and 3 only
(b) 2 and 4 only
(c) 1 and 3 only
(d) 1, 2, 3 and 4

ANSWER: C
• Manganese: Manganese is an important raw material for smelting of iron ore and also used for
manufacturing ferro alloys. Balaghat Mine is one of the largest mine producing one of the best quality of
manganese ore in the country. It is also the deepest underground manganese mine in Asia. About 80% of
the manganese production in India comes from Balaghat District. Hence, pair 4 is not correctly matched.
• Aluminum: Bauxite is the ore that is used in the manufacturing of aluminum. Bauxite is found mainly in
tertiary deposits and is associated with laterite rocks occurring extensively either on the plateau or hill
ranges of peninsular India and also in the coastal tracts of the country.
• The state of Odisha is the largest producer of bauxite ore in India. More than 95% of the bauxite resources
of the state come under East Coast Bauxite. Most of the aluminum production comes from the
Panchpatmali hills mine operated by NALCO. Hence, pair 2 is not correctly matched.
• The patlands of Jharkhand in Lohardaga have rich deposits. Gujarat, Chhattisgarh, Madhya Pradesh and
Maharashtra are other major producers.
• Iron: India is endowed with fairly abundant resources of iron ore. It has the largest reserve of iron ore in
Asia. The two main types of ore found in our country are haematite and magnetite. The iron ore mines occur
in close proximity to the coal fields in the north-eastern plateau region of the country which adds to their
advantage.
❖ About 95 percent of total reserves of iron ore is located in the States of Odisha, Jharkhand, Chhattisgarh,
Karnataka, Goa, Telangana, Andhra Pradesh and Tamil Nadu. The important mines are Gurumahisani,
Sulaipet, Badampahar (Mayurbhaj), Kiruburu(Kendujhar) and Bonai (Sundergarh). Hence, pair 3 is correctly
matched.
• Copper: Copper is an indispensable metal in the electrical industry for making wires, electric motors,
transformers and generators. It is alloyable, malleable and ductile. It is also mixed with gold to provide
strength to jewelry. The major copper mines are the Khetri copper belt in Rajasthan, Singhbhum copper
belt in Jharkhand and Malanjkhand copper belt in Madhya Pradesh. Hence, pair 1 is correctly matched.

Geography- Medium
64. Which among the following form the parts of Continental Margin?
1) Continental Shelf
2) Deep Oceanic Trenches
3) Mid Oceanic Ridges
4) Abyssal Plains
Select the correct answer using the codes given below.
(a) 1 and 2 only
(b) 1 and 4 only
(c) 1, 2 and 3 only
(d) 2, 3 and 4 only
www.civilstaphimachal.com Contact: 7814622609

ANSWER: A
• Continental Margins include- continental shelf, continental slope , continental rise and deep oceanic
trenches.
• Abyssal plains are extensive plain areas that lie between the continental margins and mid oceanic ridges.
• Mid oceanic ridges are interconnected chain of mountain system that lie in within ocean.

Geography - Easy
65. The union territory of Puducherry is surrounded by which of the following states?
1) Tamil Nadu
2) Kerala
3) Andhra Pradesh
Select the correct answer using the code given below.
(a) 1 only
(b) 1 and 2 only
(c) 2 and 3 only
(d) 1, 2 and 3

ANSWER: D
• The Union Territory of Puducherry comprises the former French establishments of Puducherry, Karaikal,
Mahe and Yanam, which lie scattered in South India. Puducherry, the capital of the Territory was once the
original headquarters of the French in India, is situated on the Coromandel Coast of the Bay of Bengal. It is
bounded on the east by the Bay of Bengal and on the three sides by Tamil Nadu.
• South of Puducherry on the East Coast lies Karaikal.
• Mahe is situated on the Malabar coast on the Western Ghats surrounded by Kerala.
• Yanam is situated adjoining the East Godavari district of Andhra Pradesh.
www.civilstaphimachal.com Contact: 7814622609

Geography - Easy
66. Polje is a
(a) Type of rice crop
(b) Moraine found in glacial landscape
(c) Large depressions in a karst topography region
(d) Another name for ocean trenches

ANSWER: C
• In Yugoslavia, very large depressions called Polje are found. These are characteristic features of a karst
topographic region.

History- Tough
67. Consider the following statements about Jagjeevan Ram:
1. He was a member of the Constituent Assembly, which framed the constitution.
2. He was also known as Deendayal.
3. He was also awarded the Victoria Medal by the then Viceroy for his services in the Chin-Lushai Expedition
of 1889-90.
Which of the statements given above are correct?
(a) 1 and 2 only
(b) 2 and 3 only
(c) 1 and 3 only
(d) 1, 2 and 3

ANSWER: C
www.civilstaphimachal.com Contact: 7814622609

• Jagjivan Ram was a towering political personality, a crusader of social justice, an exceptionally gifted orator
and a true democrate, who holds a world record of being a parliamentarian uninterrupted for a record 50
years between 1936 and 1986. Jagjivan Babu was also a member of the Constituent Assembly, which framed
the constitution.
• Also known as Babuji, Jagjivan Ram’s oratory won him the favour of Pandit Madan Mohan Malviya, when
the founder of Akhil Bharatiya Hindu Mahasabha visited his school. Impressed by his welcome address,
Malviya invited him to join the Banaras Hindu University, which proved to be a turning point in his life.
• He was a freedom fighter, a champion of depressed classes, an outstanding parliamentarian and an able
administrator. With his vast understanding of Indian social and political system and issues, he remained very
relevant in Indian politics till his last with his impeccable commitment, dedication and devotion to the
society.
• Jagjivan Ram imbibed his idealism, humanitarian values and political and social resilience from his father,
who was of deep religious disposition and the Mahant of the Shiv Narayani sect, besides being a renowned
medical practitioner, who had been in the British army and was also awarded the Victoria Medal by the then
Viceroy for his services in the Chin-Lushai Expedition of 1889-90.

History- Medium
68. Which of the following Resolutions were adopted by Congress at Karachi Congress Session of 1931?
(a) The Delhi Pact or Gandhi-Irwin Pact was endorsed.
(b) The goal of purna swaraj was reiterated.
(c) Two resolutions were adopted—one on Fundamental Rights and the other on National Economic
Programme.
(d) All of the above

ANSWER: D
About Congress Resolutions at Karachi Congress Session of 1931
• While disapproving of and dissociating itself from political violence, the Congress admired the ‘bravery’
and ‘sacrifice’ of the three martyrs.
• The Delhi Pact or Gandhi-Irwin Pact was endorsed.
• The goal of purna swaraj was reiterated.
• Two resolutions were adopted—one on Fundamental Rights and the other on National Economic
Programme— which made the session particularly memorable. The Resolution on Fundamental Rights
guaranteed—
• Free speech and free press
• Right to form associations
• Right to assemble
• Universal adult franchise
• Equal legal rights irrespective of caste, creed and sex
• Neutrality of state in religious matters
• Free and compulsory primary education
• Protection to culture, language, script of minorities and linguistic groups
• The resolution on national economic programme included—
• Substantial reduction in rent and revenue in the case of landholders and peasants
• Exemption from rent for uneconomic holdings
• Relief from agricultural indebtedness
• Control of usury
• Better conditions of work including a living wage, limited hours of work and protection of women
• Workers in the industrial sector
• Right to workers and peasants to form unions
• State ownership and control of key industries, mines and means of transport
• This was the first time the congress spelt out what swaraj would mean for the masses—”in order to end
exploitation of masses, political freedom must include
www.civilstaphimachal.com Contact: 7814622609

• Economic freedom of starving millions.”


• The karachi resolution was to remain, in essence, the basic political and economic programme of the
congress in later years.

History- Easy
69. Who among the following had set up the Anti-Untouchability League for the eradication of the practice
of untouchability?
(a) Dr. B.R. Ambedkar
(b) Mohandas Karamchand Gandhi
(c) Jyotibha Phule
(d) Motilal Nehru

ANSWER: B
About Anti-Untouchability League
• Gandhiji founded the All India Anti-Untouchability League in September 1932 in the wake of Gandhiji’s Epic
Fast at Yervada Jail, Pune, resulting in the historic Poona Pact
• It was also known as the All India Harijan Sevak Sangh and was an independent national-level non-profit
voluntary Organisation.
• It was established to combat untouchability and a new weekly paper, the Harijan, was started
• Since its inception, the organisation has helped the backward classes in many ways.
• The Sangh helped the depressed classes to access public places such as temples, schools, roads and water
resources, also conducted inter dining and inter-caste marriages.

History- Easy
70. Which among the following personalities are associated with underground Activity during Quit India
Movement?
1. Rammanohar Lohia
2. Jayaprakash Narayan
3. Aruna Asaf Ali
4. Usha Mehta
5. Biju Patnaik
Select the correct answer using the codes given below:
(a) 1, 2, 3 and 4 only
(b) 2, 3, 4 and 5 only
(c) 3, 4 and 5 only
(d) 1, 2, 3, 4 and 5

ANSWER: D
About underground Activity during Quit India Movement
• Many nationalists went underground and took to subversive activities. The participants in these activities
were the Socialists, Forward Bloc members, Gandhi ashramites, revolutionary nationalists and local
organisations in Bombay, Poona, Satara, Baroda and other parts of Gujarat, Karnataka, Kerala, Andhra,
United Provinces, Bihar and Delhi.
• The main personalities taking up underground activity were Rammanohar Lohia, Jayaprakash Narayan,
Aruna Asaf Ali, Usha Mehta, Biju Patnaik, Chhotubhai Puranik, Achyut Patwardhan, Sucheta Kripalani and
R.P. Goenka.
• Usha Mehta started an underground radio in Bombay.
• This phase of underground activity was meant to keep up popular morale by continuing to provide a line of
command and guidance to distribute arms and ammunition.

History- Easy
71. Which of the statements about proposals set by the Rajagopalachari Formula is/are correct?
1. Muslim League to endorse Congress demand for independence.
www.civilstaphimachal.com Contact: 7814622609

2. After the end of the war, the entire population of Muslim majority areas in the North-West and North-East
India to decide by a plebiscite, whether or not to form a separate sovereign state.
Select the correct answer using the codes given below:
(a) 1 only
(b) 2 only
(c) Both 1 and 2
(d) Neither 1 nor 2

ANSWER: C
About Rajagopalachari Formula
Meanwhile, efforts were on to solve the ongoing constitutional crisis, and some individuals also tried to come
up with constitutional proposals.
The Formula
C. Rajagopalachari (CR), the veteran Congress leader, prepared a formula for Congress-League cooperation in
1944. It was a tacit acceptance of the League’s demand for Pakistan. Gandhi supported the formula. The main
points in the CR Plan were:
1. Muslim League to endorse Congress demand for independence.
2. League to cooperate with Congress in forming a provisional government at centre.
3. After the end of the war, the entire population of Muslim majority areas in the North-West and North-East
India to decide by a plebiscite, whether or not to form a separate sovereign state.
4. In case of acceptance of partition, agreement to be made jointly for safeguarding defence, commerce,
communications, etc.
The above terms to be operative only if England transferred full powers to India.
Objections
Jinnah wanted the Congress to accept the two-nation theory.
He wanted only the Muslims of North-West and North-East to vote in the plebiscite and not the entire
population.
He also opposed the idea of a common centre.
While the Congress was ready to cooperate with the League for the independence of the Indian Union, the
League did not care for independence of the Union.
It was only interested in a separate nation.
Hindu leaders led by Vir Savarkar condemned the CR Plan.

History- Easy
72. Which of the following is/are the main points of Attlee’s Statement of February 20, 1947?
1. A deadline of June 30, 1948 was fixed for transfer of power even if the Indian politicians had not agreed by
that time on the constitution.
2. The British would relinquish power either to some form of central government or in some areas to the
existing provincial governments if the constituent assembly was not fully representative, i.e., if the Muslim
majority provinces did not join.
3. British powers and obligations vis-a-vis the princely states would lapse with transfer of power, but these
would not be transferred to any successor government in British India.
Which of the statements given above is/are correct?
(a) 1 and 2 only
(b) 2 and 3 only
(c) 1 and 3 only
(d) 1, 2 and 3

ANSWER: D
About Attlee’s Statement of February 20, 1947
Clement Attlee, the British prime minister, sensing the trouble all around, made an announcement on February
20, 1947. The British House of Commons declared the British intention of leaving the Indian subcontinent.
Main Points of Attlee’s Statement
www.civilstaphimachal.com Contact: 7814622609

1. A deadline of June 30, 1948 was fixed for transfer of power even if the Indian politicians had not agreed by
that time on the constitution.
2. The British would relinquish power either to some form of central government or in some areas to the
existing provincial governments if the constituent assembly was not fully representative, i.e., if the Muslim
majority provinces did not join.
3. British powers and obligations vis-a-vis the princely states would lapse with transfer of power, but these
would not be transferred to any successor government in British India.
4. Mountbatten would replace Wavell as the viceroy.
The statement contained clear hints of partition and even Balkanisation of the country into numerous states
and was, in essence, a reversion of the Cripps Offer.

History- Medium
73. Who among the following was the founder of the Saka era which starts from 78 A.D.?
(a) Chandragupta Maurya
(b) Milinda
(c) Vashishtaputra Pulamayi
(d) Kanishka

ANSWER: D
About Kanishka (78 – 120 A.D.)
• Kanishka was the most important ruler of the Kushana dynasty.
• He was the founder of the Saka era which starts from 78 A.D. Hence, option d is the correct answer.
• He was not only a great conqueror but also a patron of religion and art.

History- Medium
74. Which of the following formed part of the major exports of the Sangam age?
1. Pepper
2. Ginger
3. Ivory products
4. Gold
Select the correct answer using the codes given below:
(a) All except 1
(b) All except 2
(c) All except 3
(d) All except 4

ANSWER: D
About trade during Sangam Age
• The port city of Puhar became an important place of foreign trade, as big ships entered this port containing
precious goods.
• Other significant ports of commercial activity were Tondi, Musiri, Korkai, Arikkamedu and Marakkanam.
• Many gold and silver coins that were issued by the Roman Emperors like Augustus, Tiberius and Nero have
been found in all parts of Tamil Nadu indicating flourishing trade.
• Major exports of the Sangam age were cotton fabrics and spices like pepper, ginger, cardamom, cinnamon
and turmeric along with ivory products, pearls and precious stones.
• Major imports for the traders were horses, gold, and sweet wine.

History- Medium
75. Pataliputra is situated at the confluence of which of the following rivers?
1. Ganga
2. Yamuna
3. Son
4. Brahmaputra
www.civilstaphimachal.com Contact: 7814622609

Select the correct answer using the codes given below:


(a) 1 and 2 only
(b) 2 and 3 only
(c) 1 and 3 only
(d) 2 and 4 only

ANSWER: C
About Pataliputra
• Udayin laid the foundation of the new capital at Pataliputra situated at the confluence of the two rivers, the
Ganges and the Son.
• Later it became famous as the imperial capital of the Mauryas.

History- Tough
76. In the context of Later Vedic period, who was a Sangrihiti?
(a) Officer for collection of taxes and tributes
(b) Priests who performed Ashvamedha.
(c) Person incharge of military affairs.
(d) Officer who looked after the day to day administration.

ANSWER: A
About Sangrihitri
In the later Vedic age, collection of taxes and tributes was made compulsory and was done by Sangrihitri.

History- Medium
77. Which of the following are two important assemblies of Vedic Period?
1. Sabha
2. Mahasabha
3. Samiti
4. Ur
Select the correct answer using the codes given below:
(a) 1 and 2 only
(b) 1 and 3 only
(c) 2 and 3 only
(d) 2 and 4 only

ANSWER: B
About Assemblies of Vedic Period
• The term Sabha denotes both the assembly (in early Rig-Vedic) and the assembly hall (Later Rig-Vedic).
• Women who were called Sabhavati also attended this assembly.
• It was basically a kin-based assembly and the practice of women attending it was stopped in later-Vedic
times.
• The references to samiti come from the latest books of the Rig-Veda showing that it assumed importance
only towards the end of the Rig-Vedic period.
• Samiti was a folk assembly in which people of the tribe gathered for transacting tribal business.
• It discussed philosophical issues and was concerned with religious ceremonies and prayers.
• References suggest that the Rajan was elected and re-elected by the Samiti.

History- Easy
78. Who among the following is known as the ‘Indian Machiavelli’?
(a) Kautilya
(b) Visakadatta
(c) Megasthenes
(d) None of the above
www.civilstaphimachal.com Contact: 7814622609

ANSWER: A
About Kautilya
• Kautilya wrote Arthasastra in Sanskrit.
• He was a contemporary of Chandragupta Maurya.
Kautilya was also called ‘Indian Machiavelli’

Science- Tough
79. Consider the following statements about Hepatitis and its types:
1. Hepatitis A is usually a short-term infection while hepatitis B and C can cause long-term, or chronic,
infections.
2. A person cannot have both hepatitis B and hepatitis C at the same time.
3. Hepatitis C usually only spreads through blood-to-blood contact.
Which of the statements given above are correct?
(a) 1 and 2 only
(b) 2 and 3 only
(c) 1 and 3 only
(d) 1, 2 and 3

ANSWER: C
About Hepatitis and its types
• The most common types of hepatitis are A, B, and C. Hepatitis A is usually a short-term infection while
hepatitis B and C can cause long-term, or chronic, infections.
• A person can have both hepatitis B and hepatitis C at the same time. This article will examine the difference
between these two viruses, the treatment options available, and the outlook for people who have an
infection.
• Hepatitis B and hepatitis C are both viral infections that attack the liver, and they have similar symptoms.
• The most significant difference between hepatitis B and hepatitis C is that people may get hepatitis B from
contact with the bodily fluids of a person who has the infection.
• Hepatitis C usually only spreads through blood-to-blood contact.
• Neither hepatitis B nor C spreads through coughing, breast milk, sharing food with, or hugging a person who
has the infection.

Science- Medium
80. Consider the following statements about Zika virus:
1. Zika virus is transmitted primarily by Aedes mosquitoes, which bite mostly during the day.
2. Zika virus infection during pregnancy can cause infants to be born with microcephaly and other congenital
malformations as well as preterm birth and miscarriage.
Which statements given above is/are correct?
(a) 1 only
(b) 2 only
(c) Both 1 and 2
(d) Neither 1 nor 2

ANSWER: C
About Zika virus
• Zika virus is transmitted primarily by Aedes mosquitoes, which bite mostly during the day.
• Most people with Zika virus infection do not develop symptoms; those who do typically have symptoms
including rash, fever, conjunctivitis, muscle and joint pain, malaise and headache that last for 2–7 days.
• Zika virus infection during pregnancy can cause infants to be born with microcephaly and other congenital
malformations as well as preterm birth and miscarriage.
www.civilstaphimachal.com Contact: 7814622609

• Zika virus infection is associated with Guillain-Barré syndrome, neuropathy and myelitis in adults and
children.
• In February 2016, WHO declared Zika-related microcephaly a Public Health Emergency of International
Concern (PHEIC), and the causal link between the Zika virus and congenital malformations was confirmed.
WHO declared the end of the PHEIC in November of the same year.
• Although cases of Zika virus disease declined from 2017 onwards globally, transmission persists at low levels
in several countries in the Americas and other endemic regions.

Science- Medium
81. Consider the following statements about H1N1:
1. It is also called swine flu.
2. The World Health Organization (WHO) declared the H1N1 flu to be a pandemic in 2008.
Which of the statements given above is/are correct?
(a) 1 only
(b) 2 only
(c) Both 1 and 2
(d) Neither 1 nor 2

ANSWER: A
About H1N1
• The H1N1 flu, sometimes called swine flu, is a type of influenza A virus.
• During the 2009-10 flu season, a new H1N1 virus began causing illness in humans. It was often called swine
flu and was a new combination of influenza viruses that infect pigs, birds and humans.
• The World Health Organization (WHO) declared the H1N1 flu to be a pandemic in 2009. That year the virus
caused an estimated 284,400 deaths worldwide. In August 2010, WHO declared the pandemic over. But the
H1N1 flu strain from the pandemic became one of the strains that cause seasonal flu.

Science- Easy
82. Consider the following pairs:
1. Opaque object- we cannot see through an object at all
2. Transparent object- when we are able to see clearly through an object
3. Translucent object- when we can see through some objects, but not very clearly
Which of the pairs given above is/are correct?
(a) 1 and 2 only
(b) 2 and 3 only
(c) 1 and 3 only
(d) 1, 2 and 3

ANSWER: D
About opaque, transparent and translucent objects
• If we cannot see through an object at all, it is an opaque object.
• If you are able to see clearly through an object, it is allowing light to pass through it and is transparent.
• There are some objects through which we can see, but not very clearly. Such objects are known as
translucent.

Science- Medium
83. With reference to pneumoconiosis, consider the following statements:
1. It is the general term for a class of interstitial lung disease where inhalation of dust has caused interstitial
fibrosis.
2. The three most common types are asbestosis, silicosis, and coal miner's lung.
Which of the statements given above is/are correct?
(a) 1 only
(b) 2 only
www.civilstaphimachal.com Contact: 7814622609

(c) Both 1 and 2


(d) Neither 1 nor 2

ANSWER: C
About Pneumoconiosis
Pneumoconiosis is the general term for a class of interstitial lung disease where inhalation of dust (for example,
ash dust, lead particles, pollen grains etc) has caused interstitial fibrosis. The three most common types are
asbestosis, silicosis, and coal miner's lung.
Pneumoconiosis often causes restrictive impairment, although diagnosable pneumoconiosis can occur without
measurable impairment of lung function. Depending on extent and severity, it may cause death within months
or years, or it may never produce symptoms.
It is usually an occupational lung disease, typically from years of dust exposure during work in mining; textile
milling; shipbuilding, ship repairing, and/or shipbreaking; sandblasting; industrial tasks; rock drilling (subways
or building pilings); or agriculture. It is one of the most common occupational diseases in the world.

Science- Medium
84. Consider the following statements about Artificial sweeteners:
1. Aspartame is roughly 100 times as sweet as cane sugar and is more stable than Alitame.
2. Sucralose is tetrachloro derivative of sucrose.
Which of the statements given above is/are correct?
(a) 1 only
(b) 2 only
(c) Both 1 and 2
(d) Neither 1 nor 2

ANSWER: D
About Artificial Sweetners
1. Aspartame is the most successful and widely used artificial sweetener.
• It is roughly 100 times as sweet as cane sugar.
• It is methyl ester of dipeptide formed from aspartic acid and phenylalanine.
• Use of aspartame is limited to cold foods and soft drinks because it is unstable at cooking temperature.
2. Alitame is high potency sweetener, although it is more stable than aspartame, the control of sweetness of
food is difficult while using it.
3. Sucralose is trichloro derivative of sucrose.
• Its appearance and taste are like sugar. It is stable at cooking temperature. It does not provide calories.

Science- Tough
85. The structure ‘hypha’ is associated with which of the following?
(a) Algae
(b) Fungi
(c) Bacteria
(d) Virus

ANSWER: B
About hypha
• A hypha is a long, branching, filamentous structure of a fungus, oomycete, or actinobacterium. In most fungi,
hyphae are the main mode of vegetative growth, and are collectively called a mycelium.
Structure
• A hypha consists of one or more cells surrounded by a tubular cell wall. In most fungi, hyphae are divided
into cells by internal cross-walls called "septa" (singular septum). Septa are usually perforated by pores large
enough for ribosomes, mitochondria, and sometimes nuclei to flow between cells. The major structural
polymer in fungal cell walls is typically chitin, in contrast to plants and oomycetes that have cellulosic cell
walls. Some fungi have aseptate hyphae, meaning their hyphae are not partitioned by septa.
www.civilstaphimachal.com Contact: 7814622609

• Hyphae have an average diameter of 4–6 µm.

Science- Tough
86. With reference to the term caecum, consider the following statements:
1. Animals like horses, rabbit, etc., have a large sac-like structure called Caecum between the esophagus and
the small intestine.
2. The cellulose of the food is digested here by the action of certain bacteria which are not present in humans.
Which of the statements given above is/are correct?
(a) 1 only
(b) 2 only
(c) Both 1 and 2
(d) Neither 1 nor 2

ANSWER: C
About Caecum
• Animals like horses, rabbit, etc., have a large sac-like structure called Caecum between the oesophagus and
the small intestine.
• The cellulose of the food is digested here by the action of certain bacteria which are not present in humans.

Science- Tough
87. Copper Sulphate test is performed to detect the presence of which of the following in food items:
(a) Carbohydrates
(b) Fats
(c) Proteins
(d) Calcium

ANSWER: C
About detecting presence of Protein in food item
• Take a small quantity of food item for testing. If the food you want to test is a solid, you first need to make
a paste of it or powder it. Grind or mash a small quantity of the food item.
• Put some of this in a clean test tube, add 10 drops of water to it and shake the test tube.
• Now, using a dropper, add two drops of solution of copper sulphate and ten drops of solution of caustic
soda to the test tube. Hence, option c is the correct answer.
• Shake well and let the test tube stand for a few minutes.
• A violet colour indicates presence of proteins in the food item.

Science- Medium
88. Which of the following glands is the largest glands in the human body?
(a) Pancreas
(b) Liver
(c) Pituitary
(d) Thyroid

ANSWER: B
About human liver
• The liver is a reddish-brown gland situated in the upper part of the abdomen on the right side.
• It is the largest gland in the body.
• It secretes bile juice that is stored in a sac called the gall bladder.
• The bile plays an important role in the digestion of fats.

Environment - Easy
89. When number of food chains is interlocked this is called
www.civilstaphimachal.com Contact: 7814622609

(a) Food link


(b) Food chain
(c) Food web
(d) Pyramid

ANSWER: C
Food web: In general, the food chains in an ecosystem are not isolated, but are interconnected with one
another; i. e. an herbivore may feed on several species of plants, and/ or be consumed by many consumers and
so on, such a number of interconnected food chains is known as food web.

Environment - Tough
90. This Wildlife Sanctuary is spread over an area of 190 square kilometers. Alluvial grasslands form the major
area of the Sanctuary. Some of the portion of the sanctuary is the wooded area and the rest is water. The
eastern, western and northern parts is surrounded by the Siang River. The sanctuary comprises of a series
of river islands. The unique ecosystem of water bodies that exists here makes the sanctuary a home to
several migratory birds from Mongolia and Siberia.
The grassy and forested terrain of the sanctuary provides an ideal home to wild buffalo, deer, elephants
and certain other rain forest species. The lucky traveller shall catch a glimpse of the Royal Tiger as well.
The woodlands for the sanctuary make it an ideal habitat for a host of animals namely the leopard,
leopard cat, barking deer, civet cat, sambar, jackal, wild boar, python, porcupine and some others too.
Additionally, the savannah type of grasslands of the sanctuary make it a congenial habitat for the
migratory elephants.
The above description refers to which of the following Wildlife Sanctuaries?
(a) Daying Ering Wildlife Sanctuary
(b) Kaziranga Wildlife Sanctuary
(c) Dibru Saikhowa Wildlife Sanctuary
(d) Jaldapara Wildlife Sanctuary

ANSWER: A
About the Sanctuary:
The Daying Ering Wildlife Sanctuary in Arunachal Pradesh is spread over an area of 190 square kilometers.
Alluvial grasslands form the major area of the Daying Ering Sanctuary. Some of the portion of the sanctuary is
the wooded area and the rest is water. The eastern, western and northern parts of the Daying Ering Sanctuary
are surrounded by the Siang River. From the southern part the sanctuary extends upto the state of Assam. The
Daying Ering Wildlife Sanctuary is situated 13 kilometers off Pasighat. The sanctuary comprises of a series of
river islands. The unique ecosystem of water bodies that exists here makes the sanctuary a home to several
migratory birds from Mongolia and Siberia.
Flora and Fauna:
The Daying Ering Sanctuary holds important communities of animals and plants. Apart from other important
varieties pitcher plants are also found in this wildlife sanctuary.
The grassy and forested terrain of the sanctuary provides an ideal home to wild buffalo, deer, elephants and
certain other rain forest species. The lucky traveller shall catch a glimpse of the Royal Tiger as well. The
woodlands for the sanctuary make it an ideal habitat for a host of animals namely the leopard, leopard cat,
barking deer, civet cat, sambar, jackal, wild boar, python, porcupine and some others too. Additionally, the
savannah type of grasslands of the sanctuary make it a congenial habitat for the migratory elephants.
The waters of the Siang River is the major attraction of the Daying Ering Sanctuary. It catapults hordes of eco-
tourists and adventure enthusiasts.

Environment - Easy
91. Which of the following is a greenhouse gas?
1. Carbon Dioxide
2. Methane
3. Nitrous Oxide
www.civilstaphimachal.com Contact: 7814622609

4. Halocarbons
5. Molecular Hydrogen
Choose the correct answer using the codes given below:
(a) 1, 2 and 3 only
(b) 2, 3, 4 and 5 only
(c) 4 and 5 only
(d) All of the above

ANSWER: D
The Earth has a natural greenhouse effect due to trace amounts of water vapour (H2O), carbon dioxide (CO2),
methane (CH4) and nitrous oxide (N2O) in the atmosphere. These gases let the solar radiation reach the Earth’s
surface, but they absorb infrared radiation emitted by the Earth and thereby lead to the heating of the surface
of the planet. One needs to distinguish between the natural greenhouse effect and the enhanced greenhouse
effect. The natural greenhouse effect is caused by the natural amounts of greenhouse gases, and is vital to life.
In the absence of the natural greenhouse effect the surface of the Earth would be approximately 33 °C cooler.
The enhanced greenhouse effect refers to the additional radiative forcing resulting from increased
concentrations of greenhouse gases induced by human activities. The main greenhouse gases whose
concentrations are rising are carbon dioxide, methane, nitrous oxide, hydrochlorofluorocarbons (HCFCs),
hydrofluorocarbons (HFCs) and ozone in the lower atmosphere.
The Global Atmosphere Watch (GAW) observes, analyses and publishes greenhouse gas data collected by fifty
countries around the globe from the High Arctic to the South Pole. The greenhouse gases monitored include:
Carbon Dioxide (CO2)
Methane (CH4)
Nitrous Oxide (N2O)
Halocarbons and SF6
Molecular Hydrogen (H2)

Environment - Easy
92. Consider the following statements about Biofortification:
1. It is the process of improving the nutritional quality of food crops.
2. It can be achieved through agronomic practices, conventional breeding or biotechnology-based
approaches like genetic engineering and genome editing.
3. Golden Rice, named for the golden color of the beta-carotene added through genetic modification is an
example of biofortification
Which of the statements given above is/are correct?
(a) 1 and 2 only
(b) 2 and 3 only
(c) 1 and 3 only
(d) 1, 2 and 3

ANSWER: D
Biofortification is the process of improving the nutritional quality of food crops. This can be achieved through
agronomic practices, conventional breeding or biotechnology-based approaches like genetic engineering and
genome editing.
Biofortification of staple crops such as rice is intended as a sustainable, cost-effective and food-based means
of delivering target micronutrients to populations who do not have access to or cannot afford diverse diets and
other existing interventions such as fortified foods and supplementation.
Golden Rice
Golden Rice, named for the golden color of the beta-carotene added through genetic modification, is approved
for commercial propagation in the Philippines, where it is known as Malusog Rice. It has been assessed to be
as safe as ordinary rice, with the added benefit of beta-carotene in its grains by Food Standards Australia New
Zealand (FSANZ), Health Canada, the United States Food and Drug Administration (US FDA), and the Philippine
Department of Agriculture-Bureau of Plant Industry (DA-BPI).
www.civilstaphimachal.com Contact: 7814622609

Despite the success of existing nutrition interventions, Vitamin A deficiency (VAD) continues to be a leading
cause of preventable childhood blindness and increased risk of infection for over 190 million children
worldwide.
High Iron and Zinc Rice
The International Rice Research Institute (IRRI) is developing high-iron and high-zinc rice through transgenic
and conventional breeding approaches as a novel, food-based approach to complement current interventions
that aim to alleviate iron and zinc deficiencies. In 2015, IRRI scientists and collaborators were successful in
developing a proof of concept— a product delivering the desired levels of iron and zinc biofortification in milled
rice.
Globally, more than 1.6 billion people are anemic. Iron deficiency anemia (IDA) can affect productivity and
cause serious health consequences, including impaired cognitive development in children, a weakened
immune system, and increased risk of morbidity. Similarly, zinc deficiency is a major cause of stunting among
children, affecting about 165 million children under the age of five who are at risk of compromised cognitive
development and physical capability (WHO, 2016).

Environment - Medium
93. Consider the following statements about various types of algae:
1. Macro-algae, which are also known as seaweeds, are multi cellular plants that grow in fresh or salt water
2. Brown, red and green algae are types of micro-algae
3. Micro-algae are photosynthetic unicellular plants that also grow in fresh or salt water, they are classified
based on their pigmentation, arrangement(s) of their photosynthetic membranes or other morphological
features.
Which of the statements given above are correct?
(a) 1 and 2 only
(b) 2 and 3 only
(c) 1 and 3 only
(d) 1, 2 and 3

ANSWER: C
Algae are divided into two main types: micro-algae and macro-algae.
Macro-algae, which are also known as seaweeds, are multi cellular plants that grow in fresh or salt water, they
are classified based on their pigmentation, which includes three groups: brown, red and green algae.
On the other hand, micro-algae are photosynthetic unicellular plants that also grow in fresh or salt water, they
are classified based on their pigmentation, arrangement(s) of their photosynthetic membranes or other
morphological features. The four groups of micro-algae are diatoms, green algae, golden algae and blue-green
algae (cyanobacteria).

Environment - Tough
94. Wallace and Weber line are imaginary dividers used to mark the difference between species found in
which of the following?
(a) Australia and Papua New Guinea and Southeast Asia
(b) Africa and Asia
(c) Australia and Pacific Ocean
(d) Australia and Papua New Guinea and South America

ANSWER: A
About Wallace and Weber line
• The Wallace and Weber lines are imaginary dividers used to mark the difference between species found in
Australia and Papua New Guinea and Southeast Asia.
• This is especially apparent when considering the difference in mammals between the two regions.
• South and east of the line marsupials and monotremes are found, unlike the placental mammals found north
and west of it.
www.civilstaphimachal.com Contact: 7814622609

Environment - Medium
95. Presence of ozone in the atmosphere is important because it absorbs:
(a) Ultraviolet – A and ultraviolet – B radiations
(b) Ultraviolet – B radiations only
(c) Infrared-B radiations only
(d) Outgoing ultraviolet-B radiations and incoming ultraviolet-A radiations

ANSWER: B
UV-B: This radiation has wavelengths between 290 and 320 nanometers. UV-B radiation has been identified as
hazardous, and is primarily responsible for sunburns. UV-B radiation is mostly absorbed by the ozone layer.
Ultraviolet – A is not absorb by ozone layer

Environment - Tough
96. The National Green Tribunal, act up to deal with the expeditious disposal of cases relating to
environmental protection, was set up under which of the following?
(a) 12th five-year plan
(b) 11th five-year plan
(c) 10th five-year plan
(d) 9th five-year plan

ANSWER: B
NGT set up under The National Green Tribunal Act, 2010 in 2010.
11th FYP operated between 2007-2012.

Environment - Tough
97. Which one of the following tiger reserves of India has ‘Bhoorsingh the Barasingha’ as its official mascot?
(a) Nameri tiger reserve
(b) Ranthambhore tiger reserve
(c) Panna tiger reserve
(d) Kanha tiger reserve

ANSWER: D
Bhoorsingh the Barasingha is the official mascot of Kanha National Park. Kanha tiger reserve is now the first in
India to have an official mascot – 'Bhoorsingh the barasingha'.

Environment - Tough
98. Bio-Medical Waste Management Rules, 2016 are applicable to waste generation in which of the following?
www.civilstaphimachal.com Contact: 7814622609

1. Pathological laboratories
2. Vaccination camps
3. Ayush hospitals
4. Radioactive wastes
Select the correct answer using the codes given below;
(a) 1 and 2 only
(b) 1, 2 and 3 only
(c) 1, 2 and 4 only
(d) 1, 2, 3 and 4

ANSWER: B
About application of Bio-Medical Waste Management Rules, 2016
(1) These rules shall apply to all persons who generate, collect, receive, store, transport, treat,
dispose, or handle bio medical waste in any form including hospitals, nursing homes, clinics,
dispensaries, veterinary institutions, animal houses, pathological laboratories, blood banks, ayush hospitals,
clinical establishments, research or educational institutions, health camps, medical or surgical
camps, vaccination camps, blood donation camps, first aid rooms of schools, forensic laboratories and
research labs.
(2). These rules shall not apply to,-
(a) radioactive wastes as covered under the provisions of the Atomic Energy Act, 1962(33 of 1962) and the
rules made there under;
(b) hazardous chemicals covered under the Manufacture, Storage and Import of Hazardous Chemicals Rules,
1989 made under the Act;
(c) solid wastes covered under the Municipal Solid Waste (Management and Handling) Rules, 2000 made under
the Act;
(d) the lead acid batteries covered under the Batteries (Management and Handling) Rules, 2001 made under
the Act;
(e) hazardous wastes covered under the Hazardous Wastes (Management, Handling and Transboundary
Movement) Rules, 2008 made under the Act;
(f) waste covered under the e-Waste (Management and Handling) Rules, 2011 made under the Act; and
(g) hazardous micro-organisms, genetically engineered micro-organisms and cells covered under the
Manufacture, Use, Import, Export and Storage of Hazardous Microorganisms, Genetically Engineered
Microorganisms or Cells Rules, 1989 made under the Act.

Environment - Tough
99. Which of the following is not a regional office of United Nations Office for Disaster Risk Reduction
(UNDRR)?
(a) Nairobi
(b) Panama City
(c) Brussels
(d) Egypt

ANSWER: D
About United Nations Office for Disaster Risk Reduction (UNDRR)
• The United Nations Office for Disaster Risk Reduction (UNDRR) was created in December 1999 to ensure the
implementation of the International Strategy for Disaster Reduction.
• UNDRR (formerly UNISDR) is part of the United Nations Secretariat and it supports the implementation &
review of the Sendai Framework for Disaster Risk Reduction adopted by the Third UN World Conference
on Disaster Risk Reduction on 18 March 2015 in Sendai, Japan.
• UNDRR is led by a United Nations Special Representative of the Secretary-General for Disaster Risk Reduction
(SRSG) and has over 100 staff located in its headquarters in Geneva, Switzerland, 5 regional offices (Africa:
Nairobi, the Americas: Panama City, Arab States: Cairo, Asia-Pacific: Bangkok and Europe: Brussels).
www.civilstaphimachal.com Contact: 7814622609

• UNDRR coordinates international efforts in Disaster Risk Reduction (DRR) and it reports on the
implementation of the Sendai Framework for Disaster Risk Reduction. It convenes the biennial Global
Platform on Disaster Risk Reduction.

Environment - Medium
100. Consider the following statements about Central Pollution Control Board:
1. It is a statutory organisation under the Ministry of Home Affairs.
2. It was established in 1974 under the Air (Prevention and Control of Pollution) Act.
Which of the statements given above is/are correct?
(a) 1 only
(b) 2 only
(c) Both 1 and 2
(d) Neither 1 nor 2

ANSWER: D
About Central Pollution Control Board (CPCB)
• The Central Pollution Control Board (CPCB) of India is a statutory organisation under the Ministry of
Environment, Forests and Climate Change.
• It was established in 1974 under the Water (Prevention and Control of Pollution) Act.
• And later it was entrusted with functions and responsibilities under the Air (Prevention and Control of
Pollution) Act, 1981.
• It coordinates the activities of the State Pollution Control Boards by providing technical assistance and
guidance and also resolves disputes among them.

You might also like